Pharmacology

¡Supera tus tareas y exámenes ahora con Quizwiz!

Acute Gout---med?

Take "Colchicine with NSAID-- Naproxen--- reduced inflammation in ACUTE Gout!! *Colchicine----relief occurs within hours of the medication *naproxen given for acute gout, whereas allopurinol or pegloticase is given for chronic gout...."think PeglotiCASE vs ALLopurinol.....you ALL got a Case with Chronic gout"

duodenal ulcers can bleed, patient can be prescribed omeprazole. patient teaching?

Take before meals, med is less effective with food Only use 1 to 2 months due to A/E (FRACTURES + HYPOMAGNESEMIA), nausea, vomiting, headache other a/e to report: CALL doc; C.diff grows in neutral environments which can cause severe diarrhea *

Nurse administers ceftazidime to pt who has severe penicillin allergy. Nurse should identify which of following pt findings as indication that she should complete incident report? Shortness of breath pulse rate of 60

The client reports shortness of breath.A severe penicillin allergy is a contraindication for taking ceftazidime, a cephalosporin antibiotic, due to the potential for cross-sensitivity. Shortness of breath can indicate the client is developing anaphylaxis.

A nurse is assessing a client who is postoperative following an outpatient endoscopy procedure using midazolam. The nurse should monitor for which of the following findings as an indication that the client is ready for discharge?

The client's capnography has returned to baseline (gas exchange is adequate) Not passing of flatus as this is good for those who had general anesthesia *Capnography is an effective method to diagnose early respiratory depression and airway disorders, especially during sedation, leading to a reduction in serious complications (23, 24). Capnography provided more safety in monitoring patients during sedation. *In the setting of intubation, capnography is the 'gold-standard' method for confirmation of endotracheal tube (ETT) placement in the trachea; it also accurately detects ETT dislodgement and may decrease the frequency of inadvertent hypo- and hyperventilation.

A nurse is teaching a client who has a new prescription for levodopa/carbidopa for Parkinson's disease. Which of the following instructions should the nurse include? Increase intake of protein-rich foods Take this medication with food Takes 24 hrs to work

YES- Take this medication by mouth with or without food Not- Takes 24 hrs to work; takes effect within about 20 to 50 minutes NOT-Increase intake of protein-rich foods; Foods that are high in protein and that should be avoided in large servings......If you find that protein does interfere with the absorption of levodopa, do not cease eating protein unless directed to do so by your doctor. Rather, reschedule your meals and snacks around your medication schedule. Regular amounts of these protein rich foods may be taken 1 hour before or 1 hour after your scheduled medication time. Not-Increase intake of protein-rich foods; YES- Take this medication by mouth with or without food Note:------some say take ........Carbidopa/Levodopa Must be Taken on an Empty Stomach; To ensure that levodopa passage across the blood-brain barrier is not compromised, patients should be advised to take their carbidopa/levodopa doses an hour or more before, and 2 or more hours after eating.

Psyllium husk

bulk forming laxative-----The soluble fiber found in psyllium husks can help lower cholesterol. Psyllium can help relieve both constipation and diarrhea, and is used to treat irritable bowel syndrome, hemorrhoids, and other intestinal problems. Psyllium has also been used to help regulate blood sugar levels in people with diabetes.

diabetes insipidus

hyposecretion of ADH *treated with a synthetic hormone called desmopressin (DDAVP, Nocdurna). This medication replaces the missing anti-diuretic hormone (ADH) and decreases urination. You can take desmopressin in a tablet, as a nasal spray or by injection.

Acetominophen (Tylenol) can be given to patients w/ peptic ulcer disease b/c it does not effects blood coagulation; what meds r contraindicated?

ibuprofen NSAID naproxen NSAID aspirin NSIAD ------they all reduce platelet aggregation by reducing inflammation

herbs: feverfew

is promoted for fevers, HEADACHES*, and arthritis (anti-inflammatory); ....."Few my fever is gone and so is my headache" ***can also decrease platelet aggregation****** topically (applied to the skin), it's promoted for toothache and as an antiseptic and insecticide. Feverfew has been called "medieval aspirin" or "aspirin of the 18th century." *patient states I am having fewer headaches

Fludrocortisone is given to someone for long-term treatment for Addison's disease. What is the purpose of this therapy?

mineralocorticoids maintain electrolyte and fluid balance. Addisons disease= add more cortisol (low Na "low BP", high K, Low glucose, alopecia High levels of mineralocorticoids (Cushing disease, high Na) inc. risk of cardiac dysrhythmias due to myocardial fibrosis--Fibrotic scars of the cardiac muscle most commonly occur after myocardial infarction; however, there are various other conditions promoting cardiac fibrosis such as HYPERTENSIVE heart disease

Spironolactone (Aldactone)

potassium sparing diuretic "derived 4rm steroid"---Diuretic It can treat high blood pressure. It can also treat fluid retention (edema) and high levels of the hormone aldosterone. *Monitor increased breast tissue, gynecomastia, impotence in men, irregular menus, and hirsutism note: has been know to clear acne---trending

methotrexate

treats autoimmune diseases such as lupus, rheumatoid arthritis, + slows growth of cancer *slows growth of cancer---stops folic acid metabolism which stops cellular reproduction which are the fastest replicating cells in the body (the blood, skin, immune system, and pregnancy) A/E 1. weak immune system "suppress WBC B and T lymphocytes"= infection risk (no large crowds/ no live vaccine like herpes zooster---get flu + pneumonia vaccines that are inactivated/ report fever 100.3F/ avoid fresh fruit + flowers) 2. low platelet count 140,00-400,00= bleed risk (no razors/ do not brush teeth hard *patient must report petechia "bleeding under skin", purpura "purple spots under skin", melena "black tarry stool due to blood, hematemeis, bleeding gums 3. potential fetal death due to fast growing cells---no pregnancy must use birth control; no preggo until 1-3 months after treatment is finished

Clostridium difficile (C. diff) infection-- patient should be given?

vancomycin (antibiotic) All antibiotics are nephrotoxic Kidneys consistently filter this protein waste product out via urine. check creatinine levels (0.5-1.2)----protein in urine---muscles have normal wear and tear and must stay in an expected range. Women have lowers creatinine levels than man due to lower muscle mass

Pyridostigmine

works by preventing the breakdown of a certain natural substance (acetylCHOLINE) in your body. Acetylcholine is needed for normal muscle function. used to improve muscle strength in patients with a certain muscle disease (myasthenia gravis).

malignant hyperthermia treatment

1. Antidote Dantrolene sodium (skeletal muscle relaxant) 2-3 mg/kg -must be prompt -early detection/diagnosis 2. Stop all inhalant anesthesia agents 3. Intubate, ventilate with 100% O2 with high flow rate 4. packed in ice/cool compresses, cool IV fluids to cool temp 5. Give sodium bicarbonate for METOBOIC ACIDOSIS due to inc. metabolism

A. Patient has hypoglycemia and is conscious, nurse should admin. __________? B. Patient has hypoglycemia and is unconscious, nurse should admin. __________?

A. Oral glucose---- patient can still swallow, Evidence Based practice says for from less invasive 1st B. 50% dextrose IV ----- patient is unconscious give IV

Ondansetron (Zofran)

Antiemetic

Ondansetron (Zofran)

Antiemetic It can prevent nausea and vomiting.

diphenoxylate/atropine (Lomotil)

Atropine/ Acetylcholine/ SNS Diarrhea medication----It can treat diarrhea. *Inhibits GI motility via anticholinergic effects "SNS" *contraindicated with central angle glaucoma *structurally related to opioids so use cautiously in individuals allergic to opioids "morphine" *Too much diphenoxylate/atropine (Lomotil) can cause ileus---- ileus results from muscle or nerve problems that stop peristalsis, while an obstruction is a physical blockage in the digestive tract. *A/E- constipation, tachycardia

Glaucoma medication in the form of eye drops

Brimonidine It can treat facial redness caused by rosacea in its topical form. Other formulations can treat increased eye pressure caused by open-angle glaucoma or ocular hypertension.

A nurse is reviewing the medication list of a client who wants to begin taking oral contraceptives. The nurses should identify that which of the following client medications will interfere with the effectiveness of oral contraceptives?

Carbamazepine Anticonvulsant (cause accelerated inactivation of oral contraceptive by action on hepatic medication metabolizing enzymes)

Ciprofloxacin (Cipro)

Ciprofloxacin, a fluoroquinolone, is associated with a risk of tendon rupture. This risk is increased in older adult clients, so the client should notify the provider at the onset of tendon pain or swelling---Fluoroquinolone Antibiotic usually used to treat UTI infections along with pain med------ Phenazopyridine has been known to cause suicidal ideations, anxiety, and depression

Nurse is caring for pt who reports lethargy and myalgia after taking clozapine for months. Which of following actions should nurse plan to take?

Client is a risk for risk for agranulocytosis Obtain WBC with absolute neutrophil count ----must monitor the WBC with absolute neutrophil count weekly for the first 6 months of treatment is recommended. ----After 6 months, monitoring can occur every 2 weeks up to 1 year. ---can develop lethargy and myalgia "muscle pain" caused by the adverse effect of agranulocytosis (low RBC). "think of Schizophrenic person "ChlorPROMaZine" at prom with Amantadine "Amanda" as his date who is his antidote and decreases his extrapyradimal effects; Chlorpramazine has a twin who also has extrapyradimal effects named Clozapine who also has Schizophrenia and likes to hide in the cloZet..... he is severely sick (agranulocytosis, anticholinergic effects, high cholesterol, diabetes mellitus, Myocarditis (has a big heart), Oculogyric crisis "rolling back of the eyes", Hormonal side effects)"

Alosetron (Lotronex)

Diarrhea medication----inc. firmness of the stool and lowers sense of urgency, It can treat irritable bowel syndrome (IBS). A/E==black box warning constipation OR ischemic colitis "bowel obstruction"---SIGNED AGREEMENT IS REQUIRED due to A/E

Other Benzodiazepines for anxiolytics Give class and prototype...............

Diazepam (Valium) Lorazapam (Ativan) Alprazolam (Xanax) CHLORDIAZEPOXIDE "used for alcohol withdrawal" alcoholic having withdraws drinking PEROXIDE Chlorazepate Clonazepam (klonopin) Oxazepam midazolam (versed) Temazepam

Tumor necrosis factor (TNF)

Etanercept---intercepts "blocks" the immune response Infliximab, Adalimumab----Mab= Mad immunisupression *reported elevated WBC's, Fever 100.3F, tuberculosis activation "negative TB test needed before starting therapy/ if positive patient must go through anti-TB meds"---- *No live vaccines "no herpes booster or shingles" *Take yearly flu vaccine *DO NOT take if have infection *Labs- Report elevated WBC's--priority or Elevated CRP- C- reactive protein "checks for inflammation" not priority b/c it is expected in those with autoimmune disease (lupus, rheumatoid arthritis, psoriasis)

A nurse is preparing to admin a med to a client who has absence seizures. The nurse should anticipate administering which of the following meds to the client? Select all that apply

Ethosuximide, Valproic acid, Lamotrigine

Cancer patient is taking morphine for pain. They have started experiencing break through pain, give patient?

Fentanyl given intranasal for break through pain

Dobutamine Hydrochloride (Dobutrex)

INOTROPE-- acting on Beta-1---improves cardiac output It can treat heart failure and help the heart pump blood. For SHORT-term use in patients with decreased contractility due to heart failure or cardiac surgical procedures leading to cardiac decompensation.

Sirolimus (Rapamune) OR Everolimus

Immunosuppressive drug It can prevent organ rejection after a kidney transplant. *avoid grapefruit juice it inhibits metabolism go the drug increasing the amount in the system increasing toxicity

Nurse is caring for pt who has HF and is receiving IV infusion of dopamine. Which of following findings indicates that med is effective?

Increased cardiac outputDopamine is an adrenergic that causes a receptor specificity effect, which increases cardiac output and improves perfusion.Tachycardia is an adverse effect of dopamine and does not indicate the medication's effectiveness.Dopamine is an adrenergic that causes a receptor specificity effect, which increases blood pressure.Dopamine does not affect serum potassium levels.

A nurse is caring for a client who is receiving filgrastim. Which of the following findings should the nurse document to indicate the effectiveness of the therapy?

Increased neutrophil count (bone marrow stiumulated)----used for patients receiving chemotherapy to reduce risk of infection

Nure is providing teaching to newly licensed nurse about metoclopramide. Nurse should include in teaching that which of following conditions is contraindicated to this med?

Intestinal obstruction------Metoclopramide reduces nausea and vomiting by "increasing gastric motility" and promoting gastric emptying. "Me too clop clop clop" It is contraindicated for a client who has an intestinal obstruction or perforation.

Nurse is assessing pt who is taking tamoxifen to treat breast cancer. Which of following findings is priority for nurse to report to provider?

Leg tenderness---- Tamoxifen decreases estrogen which inc. risk for clots! The greatest risk to this client is the development of a thromboembolism, which is an adverse effect of tamoxifen. The nurse should also monitor the client for other manifestations of a thromboembolism, including leg tenderness, redness, swelling, and shortness of breath.

what follows glucose into cell when given insulin?

Potassium

client has a laceration so nurse admins epinephrine with lidocaine, why?

epinephrine causes peripheral vasoconstriction and stop systemic absorption of lidocaine

Meds for Depressive Disorders · TCAs: Amitriptyline* · SSRIs: Paroxetine*, sertraline, citalopram, escitalopram, FLUOXETINE*, vilazodone · SNRIs: Venlafaxine*, duloxetine, desvenlafaxine · MAOIs: Phenelzine*

· TCAs: Amitriptyline* Ami be tripping cause she be so depressed, hence anti-depressant.... also anticholinergic · SSRIs: Paroxetine*, sertraline, citalopram, escitalopram, fluoxetine, vilazodone · SNRIs: Venlafaxine*, duloxetine, desvenlafaxine · MAOIs: Phenelzine*

Metformin (Glucophage)

"Use: Type 2 diabetes mellitus Class: Antidiabetic, biguanide AEs: Lactic acidosis---s/s include* MYALGIA "muscle pain", malaise, somnolence, hyperventilation--patient must report finding to provider 1. can cause diarrhea

Chlorpromazine (Thorazine)

- ChlorPROMaZine is used to treat psychotic disorders such as schiZophrenia or manic-depression, and severe behavioral problems in children ages 1 through 12. -A/E orthostatic hypotension, palpitations, sedation, constipation, photosensitivity - This medication can cause photophobic skin reactions and damage to the retina of the eye if exposed to direct sunlight. PTs taking this should be reminded to wear protective clothing, apply sunscreen and wear sunglasses when they are outside. -Tardative dyskinesia is an A/E with rare neurological syndrome that has no cure---tougue moving like a worm is an example "think of Schizophrenic person "ChlorPROMaZine" at prom with Amantadine "Amanda" as his date who is his antidote and decreases his extrapyradimal effects.....he rushed upstairs to grab sunscreen + shades cause his eyes and skin is sensitive to UV; Chlorpramazine has a twin who also has extrapyradimal effects named Clozapine who also has Schizophrenia and likes to hide in the cloZet..... he is severely sick (agranulocytosis, anticholinergic effects, high cholesterol, diabetes mellitus, Myocarditis (has a big heart), Oculogyric crisis "rolling back of the eyes", Hormonal side effects)"

Anaphylaxis "sever allergic reaction" what should be administered? includes swelling eye lids, wheezing

1. 1ST line= epinephrine (constricts blood vessels "peripheral vasoconstriction)---hypotension occurs anaphylaxis, inc. cardiac output, dilates bronchiole passage---Given Sub. q, IV, or endotracheal tube 2. 2nd line= Diphenhydramine "Benadryl" decreases swelling + urticaria given IV, IM other others: *dopamine---inc. blood pressure *oral corticosteroids---decreases allergy sensitivity during re-exposure

Name what each stands for: 1. BID 2. TID 3. QID 4. Q8h

1. 2x a day "BI=2x" 2. 3x a day "Tripple" 3. 4x a day "Quadruple" 4. Every 8 hrs

Rules for injecting insulin

1. Injects injection site in SAME area--- do not rotate between the abdomen and thigh b/c absorption rate varies with the site of injection 2. DO NOT shake NPH "cloudy insulin", merely palm roll 3. Disgard Regular insulin if it is could, ONLY NPH can be cloudy

Lidocaine (Xylocaine)

1. It can treat irregular heartbeats (arrhythmias). 2. It can also relieve pain and numb the skin.

What pain relieving drugs should be given to cancer patients--- give order?

1. Naproxen "NSAID" 2. Neuropathic pain "gabapentin" 3. Opiod for moderate pain "oxycodone"

What lab values should nurse monitor when patient is taking valproic acid?

1. PT/PTT----drug can alter coagulation "bleed risk" 2. AST/ALT----drug is hepatotoxic

Fentynyl patch

1. patient who is opioid tolerant can be prescribed a patch to manage pain 2. patient with persistent pain (not intermittent) should be given patch 3. Not for post-op patients after surgery, give them less powerful pain med.

Fluticasone dry powder inhaler can be used for asthma patients. Patient teaching?

1. rinse mouth after med. admin as corticoid steroide can increase risk of oral candidiasis (thrush) 2. DO NOT shake med prior to admin but

A nurse is caring for a client who has peptic ulcer disease and reports a headache. Which of the following medications should the nurse plan to administer? ibuprofen naproxen aspirin Acetominophen

Acetominophen (Tylenol)

Nurse is assessing pt who has new prescription for chlorpromazine to Tx schizophrenia. Pt has mask-like facial expression and is experiencing involuntary movements and tremors. Which of following meds should nurse anticipate administering?

Amantadine think of Schizophrenic person at prom with Amantadine "Amanda" for their date as their antidote when having extrapyramidal s/s; The client is experiencing parkinsonism, which is an adverse effect of the antipsychotic medication chlorpromazine. Amantadine is an antiparkinsonian medication used to treat the extrapyramidal manifestations that can occur with chlorpromazine therapy.

Tobramycin

Aminoglycoside--- peak levels 5-10mcg/mL

Doxycycline Drug class: Tetracycline

Antibiotics It can treat acne, skin infections, helicobacter pylori A/E- EYES---Photosensitivity*** avoid during pregnancy teeth discoloration attacks good + bad bacteria allow C.diff to manifest or staph infection causing diarrhea---report provider

Ezetimibe (Zetia)

Antihyperlipidemic----EZE stop eating you have a lot of cholesterol blocks the absorption of cholesterol medication in the intestines may cause diarrhea due to osmosis of cholesterol in gut attracting water usually used if statin cause issue for patient or used tighter A/E- *HEPATOTOXIC--may cause hepatitis--look 4 jaundice *myopathy *pancreatitis *thrombocytopenia

Doxazosin was prescribed to a patient with benign prostatic hyperplasia

Antihypertensive drug "vasodilator" and Urinary retention medication Promote vasodilation!!!!

Clopidogrel (Plavix)

Antiplatelet---- monitor for bleeding clop clop clop grel "girl"--- this time its bleeding...period; meteclopromide----clop clop feces

A nurse is planning to administer first dose of captopril to a client has hypertension. Which of the following medications can intensify first dose hypotension? (Select all that apply.) A. Simvastatin B. Hydrochlorothiazide C. Phenytoin D. Clonidine E. Aliskiren

B. Hydrochlorothiazide ---diuretic that decreases urine output D. Clonidine----anti-hypertensive--dry mouth E. AliskiREN--- renin inhibitor and vasodilator

What factor facilitates the effectiveness of med. admin? A. protein binding site B. Lipid Solubility C. hepatic metabolism D. slow dissolution

B. med. being lipid soluble means it can cross the cell membrane---separating the medication 4rm the blood NOT A. Protein binding sites "like albumin" restrict med. from leaving blood stream targeting areas in the body NOT C. Hepatic metabolism metabolizes most meds but meds have to cross the lipid soluble plasma memebrane 1st

A nurse is planning care for a client who is receiving mannitol via continuous IV infusion. The nurse should monitor the client for which of the following adverse effects?

Bibasilar crackles (heart failure and pulmonary edema) indicated for intraocular pressure or increased intacranial pressure ICP Mannitol is a osmotic diuretic---lung crackles is a potential complication---STOP THE INFUSION!!

Alendronate (Fosamax)

Bone health-----It can treat or prevent osteoporosis. It can also treat Paget's disease of the bone.

ipratropium bromide vs Tiotropium Bromide

Both are Anticholinergics Tiotropium bromide is an inhaled medication, taken 1x a day, to help widen the airways (bronchodilator) and is used in the management of COPD. Ipratropium bromide is also a bronchodilator but has a shorter duration of action and has to be taken SEVERAL times a day.

A nurse is planning care for a client who has hypertension and is to start taking metoprolol. Which of the following intervention should the nurse include in the plan of care?

Bradycardia is an adverse effect------Determine apical pulse prior to administering (pulse rate less than 60/min - notify provider for bradycardia)

A nurse is providing teaching to a client who has a new prescription for buspirone to treat anxiety. Which of the following information should the nurse include?

Buspirone has a low risk for physical or psychological dependence or tolerance.

oral prednisone

Corticosteroid used LONG term for asthma---not used for acute situations ---can be used for SEVERE rheumatoid arthritis; may cause delayed wound healing which is an issue for patients with arthritis as they usually need higher doses of prednisone severely slowing would healing schedule medication on alternate days to decrease A/E LIKE suppression of adrenal gland, decreased inflammation, low WBC count, SLOWS rate of growth in children DO NOT take on an empty stomach---steriods causes gastrodistress and ulcers

Chenodiol

Decreases cholesterol gallstone by inhibiting HMG-CoA reductase (decrease cholesterol synthesis), by decreasing intestinal reabsorp of cholesterol, & inhibiting secretion of cholesterol into bile -hepatoxic liver function test required

med. used to treat arterial fibulation which causes hypotension?

Diltiazem --- Calcium Channel Blocker; Client should monitor Blood Pressure "Diltiazem rhymes w/ calzium"

Nurse is caring for pt who has developed hypomagnesmia due to long-term therapy w/ lansoprazole. Nurse should monitor pt for which of following manifestations?

Disorientation The nurse should monitor the client for disorientation and confusion as manifestations of hypomagnesemia. The nurse should also assess the client for a positive Chvostek's and Trousseau's signs.

A nurse is providing teaching to a client who has a gastric ulcer and a new prescription for famotidine. Which of the following instructions should the nurse include?

H2- blocker----Report yellowing of the skin

Natrexone (opiate antagonist)

Helps relieve craving in 20% of alcoholics

Levothyroxine (Synthroid)

Hormone It can treat hypothyroidism. It can also treat an enlarged thyroid gland and thyroid cancer. calcium, iron, magnesium, and zinc compete with levothyroxine binding site-----decreasing it's absorption Patient taking calcium supplement should take levothyroxine on an empty stomach and take calcium supplement 4hrs after

A nurse is providing teaching to a client who is taking BUPROPION as an aid to quit smoking. Which of the following findings should the nurse identify as an adverse effect of the medication?

Insomnia People may become depressed when they quit smoking so they may be administered BUPROPION atypical antidepressant. It is a Antidepressant and Smoking cessation aid *Insomnia is an A/E ---- they may be prescribed zolpidem to treat insomnia----think Zolpidem is for the PM Bupropion--- atypical antidepressant has stimulant properties causing agitation, tremors, mania, and insomnia

Cognition-enhancing medication It can treat dementia associated with Alzheimer's disease.

Memantine

Aminophylline

Methylxanthines Bronchodilator*---just like Theophylline w/o A/E of Theophylline Theophylline May cause--- "Theo is a God + will see God if taking this med due to toxic effects" toxic + cause tachycardia, do not mix w/ caffeine (10-20 range)-

A nurse is teaching a client who has a new prescription for digoxin about manifestations of toxicity. Which of the following findings should the nurse include in the teaching? Constipation Nausea Wheezing Muscle rigidity

NauseaRationale: The nurse should instruct the client to monitor for and report manifestations of digoxin toxicity, such as nausea, anorexia, abdominal pain, bradycardia, and visual changes.

Isosorbide MonoNITRATE

Nitrate----used to prevent angina (chest pain) caused by coronary artery disease (heart disease). It does not work fast enough to relieve the pain of an angina attack that has already started. *Isosorbide mononitrate is a nitrate used for clients with angina. Taking isosorbide mononitrate leads to vasodilation, which can result in hypotension. The nurse should withhold the medication and notify the provider if the client's systolic blood pressure is below the expected reference range of 120/80. relaxing the blood vessels and increasing the supply of blood and oxygen to the heart while reducing its workload.

Hydromorphone (Dilaudid)

Opioid Analgesic

Meperidine hydrochloride (Demerol)

Opioid Analgesic not indicated for cancer cause it can cause sever toxic effects

Non-selective Beta blockers target Beta- 1 (heart, kidneys) & 2 (lungs, skeletal muscle, smooth muscle, ciliary body of eye, digestive system "liver" Name drug? Outcome?

PROPRANOLOL (pro can effects 1 + 2), Sotalol, Timolol lowers HR + strength of contractions lowers BP lower intra ocular pressure allows bhronchoconstriction "blocks sympathetic; peripheral vasoconstriction nervous system" may cause hyper/hypo glycemia

Pioglitazone Drug class: Thiazolidinedion

Pioglitazone increases cellular response to insulin-------treats high blood sugar levels caused by type 2 diabetes which is insulin resistant. A/E= 1. water retention "edema"---contraindicated in heart failure patients due to them already having fluid overload---drug promotes reabsorption of NA in the kidneys so as a results daily weights is required 2. inc. glucose and fat metabolism 3. bone fractures/ osteoporosis 4. inc. risk of pregnancy "ovulation" It may be used alone or with other medicines such as insulin, metformin, or sulfonylurea agents. Clients should monitor for hypoglycemia 2-4 hrs after taking med which is the peak time---6hrs is NOT the peak time! Cannot be taken with those with type 1 diabetes mellitus as it needs insulin to work--- these patients do NOT produce insulin

A nurse is caring for a 20 year old female client who has a prescription for isotretinoin for severe nodulocystic acne vulgaris. Before the client can obtain a refill, the nurse should advise the client that which of the following tests is required?

Pregnancy test (must provide two negative pregnancy tests for initial prescription and 1 negative test for refill)--- MED is teratogenic

Brompheniramine

Rhymes with antihistamine!!----"amine" Dimetapp----- first-generation antihistamine drug of the propylamine class. It is indicated for the treatment of the symptoms of the common cold and allergic rhinitis, such as runny nose, itchy eyes, watery eyes, and sneezing. *Can experience Cholinergic blockade--- dry mucus membranes (mouth, nasal passage, throat) *Take sugarless candy and stay hydrated

Nurse is teaching pt who is taking ranitidine/FAMOTIDINE for PUD "peptic ulcer disease". Which of following pt statements should nurse identify as understanding of teaching?

"I know smoking makes ranitidine less effective."The nurse should instruct the client that smoking decreases the effectiveness of ranitidine by exacerbating the ulcer manifestations.

Nurse is teaching pt who is starting to take diltiazem. Which of following statements should nurse identify as indication that pt understands teaching?

"I will check my heart rate before I take the medication." Diltiazem, a calcium channel blocker, has cardio-suppressant effects at the SA and AV nodes, which can lead to bradycardia. The client should check their heart rate before taking the medication and notify the provider if it falls below the expected reference range.

Nurse is teaching about zolpidem to pt who has insomnia. Nurse should identify that which of following pt statements indicates an understanding of teaching?

"I will use this medication for a short period of time."----think Zolpidem is for the PM Zolpidem is used for short-term treatment of insomnia. Therefore, the provider should reassess the client before refilling the prescription. The client who takes zolpidem should experience improved sleep within 2 days of starting this medication. People may become depressed when they quit smoking so they may be administered BUPROPION atypical antidepressant. It is a Antidepressant and Smoking cessation aid<<<zolpidem can be given to aid in sleep

Zileuton

-Antileukotrienes (leukotriene modifier/ Blocks conversion of arachidonic acid to leukotrienes. )---Anti-inflammatory-----It can prevent and control asthma attacks. -Hepatotoxic---MUST check ALT "liver" enzymes closely while taking asthma med zileuton

Patient teaching for metoprolol

-DO NOT abruptly stop medication, -take whole, -check apical and peripheral pulse -change positions slowly (orthostatic hypotension)

Non-live vaccines

-DTap- diphtheria, tetanus, pertusis vaccine -administered in the deltoid or thigh -consists of s series of 5 injections* with the 1st at 2 months, 2nd @ 4 months, 3rd at 6 months, 4th at 15-18 months, and 5th at 4-6 years old

Rules for mixing insulin:

*Never mix Insulin Glargine "Lantus" with any other type of insulin. *Administer the dose within 5 to 10 minutes after drawing up because the regular insulin binds to the NPH and this decreases its action. *Check the patient's blood sugar and for signs and symptoms of hypoglycemia to ensure they aren't hypoglycemic ...if patient is hypoglycemic hold the dose and notify md for further orders....usually less than 70 *Remember the mnemonic: RN (Regular "clear" to NPH "cloudy" or Humulin R "clear" to Humulin N "cloudy") Why? It prevents contaminating the vial of clear insulin with the cloudy insulin because if contaminated it can affect the action of the insulin. Why does this matter because they will be mixed in the syringe? You have 5 to 10 minutes to give the insulin mixed in the syringe before the action of the insulins are affected

clozapine vs clonidine

*clozapine= schizophrenia *clonidine= hypertension/sedative (Alpha-2-Agonist "blocks SNS" )---can cause dry mouth

Herb gingko biloba

*decreases platelet aggregation----blood thinner *may increase risk of bleeding with anticoagulants (warfarin "HOME USE", heparin "enoxaparin" HOSPITAL USE), antiplatelets (aspirin, clopidogrel "plavix"), and NSAIDs ......ging ging KO! no biloba (clot) given to enhance mental alertness and improve memory;

Isosorbide

-belongs to the group of medicines called nitrates. -used to prevent angina (chest pain) caused by coronary artery disease (heart disease)........It does not work fast enough to relieve the pain of an angina attack that has already started.

Meds for Anxiety Disorders

1. BENZodiazepine (BZD) sedative hypnotic anxiolytics: alprazolam*, diazepam, lorazepam, CHLORDIAZEPOXIDE "used for alcohol withdrawal---alcoholic having withdraws drinking PEROXIDE , clonazepam, oxazepam 2. HyDROXyzine "antihistamine --anticholingergic properties + anxiety"---- not the same as hyDRALAzine "vasodilator" 3. Atypical anxiolytic (non-BZD/nonbarbiturate anxiolytic): BUSPIRONE* buspirone ---non-sedating 4. Selective serotonin reuptake inhibitors (SSRIs): Paroxetine* PAROXETINE, sertraline, citalopram, ESCITALOPRAM*, fluoxetine, fluvoxamine 5. Serotonin-norepinephrine reuptake inhibitors (SNRIs): Venlafaxine*, duloxetine, desvenlafaxine Flumazenil---antidote for benzo's

A nurse is providing teaching to a client who has multiple sclerosis and a new prescription for methylprednisolone. Which of the following instructions should the nurse include?

1. Blood glucose levels will be monitored during therapy ( medication increase serum glucose level & require management) 2. Avoid contact with people who has known infection (medication decreased immunity putting patient at increased risk for infection) 3. Grapefruit juice can increase the effects of the medication

Bismuth subsalicylate (Pepto-Bismol)

1. Diarrhea/ Nausea 2. Heart burn----iindegesion--upset stomach coat a peptic ulcer and protect it from stomach acid. bismuth subsalicylate can kill H. pylori, H. pylori secretes an enzyme called urease, which converts the chemical urea to ammonia. The production of ammonia around H. pylori neutralizes the acidity of the stomach, making it more hospitable for the bacterium.

Can an opioid medication be served with Lorazepam?

No, Lorazepam (benzodiazepine) is a CNS depressant and should not be given with an opioid which is also a CNS depressant!

CEFotetan

2nd generation cephalosporin----remember penicillin has a cross sensitivity to cephlasporins *lowers vitamin k levels resulting in bleeds epistaxis

A client is scheduled to receive a tube feeding at 7:00, they are required to take insulin along with the gastrostomy feeding. What time should nurse plan to administer insulin lispro subQ?

6:45

when administering morphine IV bolus, nurse must monitor for Respiratory depression when?

7 min after administration; have naloxone ready!!

Can pregnant women take NSAID (Ibuprofen, indomethacin, naproxen)?

No, its category C for pregnancy (use if risks outweigh the benefits) *Acetominophen is not an NSAID

Live vaccines

>MMR measles mumps rebella >varicella---Gelatin has cross-sensitivity >hepatitis a---Children need 2 doses of hepatitis A vaccine: First dose: 12 through 23 months of age; followed by a second dose at least 6 months later---known to cause loss of appetite for a few days >Rota Virus----given in 3 doses at ages 2 months, 4 months, and 6 months

Malignant hyperthermia

A hereditary condition of uncontrolled heat production that occurs when susceptible people receive certain anesthetic drugs like Succinylcholine a muscle relaxant

Liver damage---check (AST/ALT)

ALT is an enzyme found in the liver that helps convert proteins into energy for the liver cells. When the liver is damaged, ALT is released into the bloodstream and levels increase. Aspartate transaminase (AST). AST is an enzyme that helps metabolize amino acids.

Acamprosate vs Naltrexone

Acamprosate (Abstinence) has been found to be slightly MORE efficacious in promoting Abstinence from alcohol and naltrexone slightly more efficacious in REDUCING heavy drinking and craving.

Imipenem/Cilastatin (Primaxin)

Antibiotics---"IMI Pin nem down he has seizures" It can treat bacterial infections. *SEIZURES can occur with this medication* Does Not Effect: ototoxicity,

A nurse is providing dietary teaching to a client who has a new prescription for phenelzine. Which of the following food recommendations should the nurse make? Broccoli Yogurt Cream cheese Pepperoni Pizza Bologna Sandwich

Answer - Broccoli, Yogurt and Cream Cheese are the recommendations Broccoli - Clients who take phenelzine, an MAOI, should not eat foods that contain tyramine. Broccoli does not contain tyramine Yogurt - Clients who take phenelzine, an MAOI, should not eat foods that contain tyramine. Yogurt contains little or no tyramineCream cheese - Clients who take phenelzine, an MAOI, should not eat foods that contain tyramine. Cream cheese contains little or no tyraminePepperoni Pizza - Clients who take phenelzine, an MAOI, should not eat foods that contain tyramine. Pepperoni is a fermented meat that is high in tyramine contentBologna Sandwich - Clients who take phenelzine, an MAOI, should not eat foods that contain tyramine. Bologna is a fermented meat that is high in tyramine content

What anticholingergic asthma med can be used when patient can't tolerate beta 2 agonist meds?

Anticholinergics---which stop the rest + digest---stop secretions= IPRATROPIUM (form of atropine for COPD) --short acting or TIOTROPIUM---long acting offer sugarless candy which promotes SALIVA production Acetylcholine-Cholinergic--- cool you down--- promotes rest and digest see, pee, spit, shit

Loperamide

Antidiarrheal

A nurse working in an emergency dept. Is caring for a client who has benzodiazepine toxicity due to an overdose. Which actions is the nurses priority? A- administer flumazenil B- identify the clients level of orientation C- infuse iv fluids D- prepare the client for gastric lavage

B- the first action the nurse should take when using the nursing process is to assess the client. Identifying the clients level of orientation is the priority action

A nurse is teaching a client who has a new prescription of ramelteon* The nurse should instruct the client to avoid which of the following foods while taking this medication? A.Baked potato B.Fried chicken C.Whole-grain bread D.Citrus fruits

B. Fried Chicken-----Taking ramelteon with a high-fat or heavy meal may delay the onset of sleep. For faster sleep onset, ramelteon should not be administered with or immediately after a high-fat or heavy meal<<<<slows drugs effectiveness

Patient has a dry nonproductive cough, which med. should be given? A. Mucolytic B. Expectorant C. Antitussive

C. Antitussive---- there is no mucus so stop cough

A nurse in an acute mental health facility is caring for a client who is experiencing withdrawal from opioid use and has a new prescription of clonidine. Which of the following actions should the nurse identify as the priority? A. Administer the clonidine on the prescribed schedule B. Provide ice chips at the client's bedside C. Educate the client on the effects of clonidine D. Obtain baseline vital signs

D. Assessment is the initial step if the nursing process. Obtaining the client's baseline VS is the priority nursing action

QT interval drugs

Drugs that can prolong the QT interval on the ECG, thereby creating a risk of serious dysrhythmias----- such as bradycardia Caution in those with: Low mag, K, hypotension elderly heart failure

A nurse is administering donepezil to a client who has Alzheimer's disease. Which of the following findings should the nurse report to the provider immediately?

Dyspnea (caused by bronchoconstriction due to increased Ach level)----- inc. PNS "acetylcholine" -- helps with Myasthenia graves + Alzeimers

name clot busters-- Thrombolysis, also called fibrinolytic therapy, is the breakdown of blood clots formed in blood vessels, using medication. It is used in ST elevation myocardial infarction, stroke, and in cases of severe venous thromboembolism.

Eminase (anistreplase) Retavase (reteplase) Streptase (streptokinase, kabikinase) t-PA (class of drugs that includes Activase) TNKase (tenecteplase)

Nurse is administering diazepam to pt who is having colonoscopy. Which of following actions should nurse take?

Ensure flumazenil is available to administer for toxicity management----The nurse should be prepared to administer flumazenil to reverse the effects of diazepam The nurse should monitor the client for manifestations of diazepam toxicity, such as respiratory depression "slow RR" and hypotension slow HR". When diazepam is administered IV for induction of anesthesia, the nurse should expect the client to develop the full effect of the medication in 2 min----REMEBER IV medications must have antidote near The nurse should measure the capnography level every 15 to 30 min (q.1 hours is not sufficient) until the client is awake and oriented and vital signs have returned to baseline.

cholesterol-lowering drugs

Gemfibrozil Ezetimibe (Zetia) Simvastatin (Zocor) alirocumab

A nurse is providing info to a client who has early Parkinson's disease and a new prescription for pramipexole. The nurse should instruct the client to monitor for which of the following adverse effects of this medication?

Hallucinations "The guy who has Parkinson's takes Pramipexole to decreases his tremors......but now has hallucinations from taking drug"

Naproxen (Aleve)

NSAID---non Opiod pain reliever

nitroglycerine vs nitroprusside

Nitroprusside (used in hospital) is a potent arterial and venous vasodilator. It produces more balanced arterial and venous dilation compared to nitroglycerin (which is more of a venodilator particularly at low doses).

A nurse is planning care for a client who is prescribed metoclopramide following bowel surgery. For which of the following adverse effects should the nurse monitor?

Sedation, dizziness, fatigue-----inc the PSNS--rest and digest!!

sertraline anti-depressant an anti-depressant should not be taken with?

St. John's wort for depression

Zidovudine (Retrovir)

Tx: HIV Class: antiretroviral AE: *paresthesia, *N/V, *lactic acidosis, *BONE MARROW SUPRESSION- low hemoglobin, sever anemia, neutropenia---Priority!! Teaching: monitor hemoglobin

Fexofenadine (Allegra)

antihistamine 7 year old can't take standard tablet meant for 12 years olds

S/S of patients taking too much levothyroxine?

chest pain tachycardia insomnia tremors hyperthermia heat intolerance diaphoresis

The elderlies change in parmacokenitics is due to?

decreased Renal function----excretion of medication I reduced--- glomeruli filtration is reduced

Barbiturates

drugs that depress the activity of the central nervous system, reducing anxiety but impairing memory and judgment dictate how LONG Cl- channels open--"Behind Bars for long time" has a small therapeutic index----inc. risk for toxicity causing mechanical ventilation, need for dialysis or activated charcoal to get rid of toxic amt---there is no antidote to reverse the effects!

A/E of beta 2 agonists Albuterol

inc. HR Jittery Nervous Tremors Insomnia---do not take at bedtime "Ive had my nebulizer treatment and I feel all nervous and jittery!"

Estradiol (Cream)

intravaginal estrogen used forage related vaginal atrophy

what is a contraindication in taking meds---

liver disease---can't break down drug

morphine

narcotic (opioid) drug derived from opium, used to treat severe pain PO- administer 60-90min prior to ambulation...thats the peak time of the drug.......meaning it is in full effect

Omeprazole

proton pump inhibitor

A nurse is preparing to administer a scheduled antibiotic at 0800 to a client and discovers the antibiotic is not present in the client's medication drawer. The nurse should identify that administration of the medication can occur at which of the following time periods without requiring an incident report?

0830 (30 min before/ after)---- otherwise file an incident REPORT!!---- THERAPAUTIC effects will diminish after 30min

Patient has sever IBS Irritable bowel syndrome (IBS) and is prescribed Alosetron; what should be included in the plan of care?

1. An agreement must be signed before taking drug for IBS a it can cause CONSTIPATION + BOWEL OBSTRUCTION----FDA required patient signs form before receiving drug 2. If drug is NOT effective within 1 month (NOT 1 WEEK) PATIENT can stop taking med and notify provider 3. A/E to report tachydysrhymia

A nurse is developing a teaching plan for a client who has a new prescription for simvastatin. Which of the following instructions should the nurse include in the teaching plan?

1. Report muscle pain to the provider. 2. Avoid taking the medication with grapefruit juice. 3. Expect therapy with this medication to be lifelong.

CEFtriaxone

3rd generation Cephalosporin antibiotic-----remember penicillin has a cross sensitivity to cephlasporins Patient who have an allergy to penicillin (Piperacillin) antibiotic can have a cross sensitivity reaction to cephalosporin antibiotic (ceftriaxone) cephalosporin antibiotic (ceftriaxone) DOES NOT have cross sensitivity to gentamicin (amino glycoside)

Patient in PACU has respiratory depression 9/min & difficult to arouse

A. Naloxone Naltrexone is for alcohol overdose

A nurse is providing instructions to a client who has been experiencing insomnia and has a new prescription for temazepam. The nurse should inform the client that which of the following manifestations are adverse effects of temazepam? (Select all that apply). A.Incoordination B.Hypertension C.Pruritus D.Sleep driving E.Amnesia

A. Incoordination D.Sleep driving E.Amnesia D.Sleep driving-----This medicine may cause you to do things while you are still asleep that you may not remember the next morning. It is possible you could drive a car, sleepwalk, have sex, make phone calls, or prepare and eat food while you are asleep or not fully awake. Note: Twenty-nine drugs, primarily in four classes-benzodiazepine receptor agonists and other gamma aminobutyric acid (GABA) modulators, antidepressants and other serotonergic agents, antipsychotics, and β-blockers-were identified as possible triggers for sleepwalking.

A charge nurse is planning a staff education session to discuss medications used during the care of a client experiencing Alcohol Withdrawal. Which if the following medications should the charge nurse include in the discussions? (select all that apply) A. Lorazepam B. Diazepam C. Disulfiram D. Naltrexone E. Acamprosate

A. Lorazepam B. Diazepam Both are sedatives; both are benzodiazepine used during alcohol withdrawal to decrease anxiety and reduce the risk of seizures Not Disulfiram (Adverse Effects) Not Naltrexone (Reduce cravings, like propranolol "beta blocker") Not Acamprosate (Abstinente)

A. Opioid (heroin) overdose eye manifestations? B. Opioid Withdraw *Heroin is an opioid drug made from morphine, a natural substance taken from the seed pod of various opium poppy plants

A. Overdose "heroin eyes">>>Pinpoint Pupils----Pupils that are abnormally small under normal lighting conditions B. Withdraw>>>Increased Pupil Size

A nurse is teaching a female client who has tobacco use disorder about nicotine replacement therapy. Which of the following statements by the client indicates an understanding of the teaching? A. "I should avoid eating right before I chew a piece of nicotine gum. "B. "I will need to stop using the nicotine gum after 1 year." C. "I know that nicotine gum is a safe alternative to smoking if I become pregnant." D. "I must chew the nicotine gum quickly for about 15 minutes."

A. The client should avoid eating or drinking 15 minutes prior to and while chewing the nicotine gum-----to make sure all the nicotine from the gum can get into your system. NOT--- "I know that nicotine gum is a safe alternative to smoking if I become pregnant."........The authors conclude that nicotine gum does not improve smoking cessation rates in pregnant women,Despite not reducing smoking during pregnancy, use of nicotine gum increased birth weight and gestational age

A nurse is providing instructions to a female client who has a new prescription for zoldipem. Which of the following instructions should the nurse include? A. "Notify the provider if you plan to become pregnant. "B. "Take the medication 1 hour before you plan to go to sleep. "C. "Allow at least 6 hours for sleep when taking zolpidem. "D. "To increase the effectiveness of zolpidem, take it with a bedtime snack."

A. zolpidem is pregnancy risk category C. The client should notify the provider if she plans to become pregnant.

A nurse is teaching a client who has a new prescription for captopril. Which of the following instructions should the nurse include in the teaching? A.) Monitor for a cough. B.) Hold medication for heart rate less than 60/min. C.) Take this medication with food. D.) Avoid grapefruit juice.

A.) Captopril is an ACE inhibitor used to treat hypertension. The client should monitor and report a cough and dyspnea. The client should avoid salt substitutes, as most of them are high in potassium NOT- Take this medication with food; The client should take captopril with no food (on empty stomach)

A nurse should identify that clopidogrel (plavix) is contraindicated for clients who have which of the following conditions? A.Peptic Ulcer Disease B. Myocardial infarction C.Pancreatitis D.Myastenia

A.Peptic Ulcer Disease Answer: Peptic ulcer disease: Clients who have peptic ulcer disease should not take clopidogrel because it can cause gastric bleeding.

Prednisone for Rheumatoid Arthritis

A/E 1. ATI says it does not cause immunosupression b/c rheumatoid drugs cause immunosuppression more. 2. Osteoporosis 3. Adrenal suppression 4. Gastric Ulcerations*

Meds contraindicated in pregnancy

ACE inhibitors; Amiodarone Antiepileptic "seizure" drugs........teratogenic nimodipine "antihypertensive drug".....teratogenic • Avoid methylergonovine in hypertensive/pre-eclampsia patients or patients with a history of hypertension as drug can cause hypertensive crises Avoid carboprost "tocolytic" in asthmatic patients who are pregnant>>>>>Carboprost should be avoided in patients with asthma or pulmonary disease as it can cause acute bronchoconstriction.

Carbamazepine MOA

Anticonvulsant It can treat seizures, nerve pain, and bipolar disorder.

client receiving IV GENTAMICIN 3 times a day is likely to have which Adverse effects: A. Hypoglycemia B. Proteinuria C. Nasal congestion D. Visual disturbances

B. NEPHROTOXICITY is an adverse effects of gentomycin amino glycoside *Diff. antibiotic 4rm PENICILLIN (Piperacillin) antibiotic who has an cross sensitivity reaction to CEPHALOSPORIN antibiotic (ceftriaxone)

patient has menopause and is prescribes estrogen + progestin for what reason? A. Long term use to prevent osteoporosis B. urogenital atrophy

B. Urogenital atrophy---- use of short-term treatment can help manage manifestations of menopause such as hot flashes, sweats, etc Long-term treatment have high risk factors that outweighs benefits of treating osteoporosis

A nurse is caring for a client who has a new prescription for alirocumab to reduce cholesterol. The nurse should monitor for which of the following findings as an adverse effect of alirocumab? (Select all that apply.)

B. Vasculitis D. Urticaria E. Jaundice

A nurse is providing teaching to a client who has a new prescription for clonidine to assist with maintenance of abstinence from opioids. The nurse should instruct the client to monitor for which of the following AE? A. Diarrhea B. Dry mouth C. Insomnia D. HTN

B. dry mouth is a common AE associated with clonidine (Alpha-2-Agonist "blocks SNS") use Being abstinent from opioids can cause a person to activate SNS---- we can give

Name 3 common anxiolytics? used for?

Benzodiazepines---dictate how OFTEN Cl- channels open--- "Go out in the Benz often!" Barbituates-- narrow therapeutic index inc. risk of toxicity---keeps cl- channels open longer, higher dependency Buspirone-- does not sedate "can drive the bus with Buspirone"

What calcium channel blocker is used for A fib?

Benzothiazepines (Diltiazem) Amlodipine; Nifedipine (pectoris angina/hypertension) - only minimally blocks calcium channels in the heart they prolongs cardiac conduction, depresses depolarization, vasodialates, and decreases oxygen demand of the heart

A nurse is completing a nursing history for a client who takes simvastatin. The nurse should identify which of the following disorders as a contraindication to adding ezetimibe to the client's medications? A. History of severe constipation B. History of hypertension C. Active hepatitis C D. Type 2 diabetes mellitus

C. Active hepatitis C

A nurse is collecting data from a client who is taking Gemfibrozil. Which of the following assessment findings should the nurse identify as an adverse reaction to the medication? A. Mental status changes B. Tremor C. Jaundice D. Pneumonia

C. Jaundice

Verapamil

Calcium channel blocker and Antihypertensive drug---- "think someone on the stage rapping and artist has his calcium channel blockers protecting him.... he is moving slow to prevent orthostatic hypotension....his song is about foods high in fiber so you do not get constipated.....the calcium channel blockers are fed up of the nifedipine/amlodipine phenes" It can treat high blood pressure, severe angina, and arrhythmia. Constipation is a common side effect of verapamil treatment. Increasing fluid and fiber intake can ease constipation.

A nurse is caring for a client who has cancer and is taking oral morphine and docusate sodium. The nurse should instruct the client that taking the docusate sodium daily can minimize which of the following adverse effects of morphine?

Constipation (due to morphine) *morphine common A/E is nausea also* Oral morphine takes atleast an HOUR to work

Raloxifene contraindications & A/E & Benefits

Contraindication= history of venous thrombotic events "Think of This of lady named RaloXifene who has s/s of low estrogen (hot flashes, decreased libido, vaginal dryness) but has inc. bone density and inc. risk for clots (DVT/ leg cramps) as if she had high amounts of estrogen......... The X signifies she can't have babies!" STOP med. 3 days before prolonged immobility such as surgery or travel raloxifene--- has s/s of low estrogen but inc. risk for clots and aids in bone mineral absorption as seen in estrogen!!!....... weird opposite effect 1. risk deep-vein thrombosis 2. pulmonary embolism 3. stroke Adverse Effects (signs of low estrogen) 1. inc. incidence of hot flashes, leg cramps, may decrease libido, or cause vaginal dryness-------works as a BRCA antagonist by blocking estrogen at the receptor site in breast tissue----protects against breast cancer also Additional S/S- bone mineral absorption + DVT Benefits 1. preserves bone mineral density----patient should undergo bone density scan at least q. year, also check for blood calcium levels since it promotes calcium absorption ---for maximum benefits consume foods rich in calcium and vitamin D--not enough calcium will cause parathyroid hormone to resorp it from bone. ---perform weight bearing exercises to promote bone density (30-40min walks each day) 2. does not inc. MI, actually lowers LDL cholesterol monitor liver function test with all drugs--- drugs can remain in the system longer if liver is not breaking it down adequately

lactulose vs lactose

Unlike lactose, lactulose cannot be hydrolyzed by human intestinal enzymes. Therefore lactulose is excreted by the body, it's a laxative

A nurse is caring for a client who is admitted to undergo a surgical procedure. Which of the following preexisting conditions can be a contraindication for the use of ketamine as an IV anesthetic? A.Peptic Ulcer Disease B.Breast cancer C.Diabetes mellitus D.Schizophrenia

D.Schizophrenia While ketamine has been shown to have rapid and robust antidepressant effects in numerous trials (2), it has also been associated with increased psychotic symptoms; Because of a theoretical risk of exacerbating psychosis in predisposed patients, subjects with current psychotic symptoms or a past history of psychosis are typically excluded from ketamine trials.

Cholorthiazide (Diuril)

Diuretic It can treat fluid retention and high blood pressure, reducing the risk of stroke and heart attack. Can cause water, na, k loss *--- Hypokalemia= muscle weakness, muscle cramps, dysrhythmias

Sildenafil (Viagra)

It can treat erectile dysfunction. It can also treat high blood pressure in the lungs (pulmonary arterial hypertension).

A nurse is reviewing the medical record of a client who has schizophrenia and is to start taking clozapine. Which of the following findings should the nurse identify as a contraindication for the client to receive clozapine? BP 150/87 mm Hg WBC count 2,800/mm3 Auditory hallucinations Nausea

WBC count 2,800/mm3Rationale: Clozapine can cause agranulocytosis, which can be life-threatening. Therefore, a WBC count of less than 3,000/mm3 is a contraindication for the client to receive clozapine. The nurse should withhold the medication and notify the provider of the client's WBC count. Auditory hallucinationsAuditory hallucinations are a positive manifestation of psychosis and are not a contraindication for the client to receive clozapine. Nausea is not a contraindication for the client to receive clozapine. The client can take clozapine with food to minimize gastrointestinal upset.

diabetes mellitus vs diabetes insipidus

While the terms "diabetes insipidus" and "diabetes mellitus" sound similar, they're NOT related. Diabetes mellitus — which involves high blood sugar levels and can occur as type 1 or type 2 — is common and often referred to simply as diabetes Diabetes Insipidus= low ADH 'vasopressin" in the posterior pituitary

Asthma meds given in combination-- Is it ok to give Salmeterol Bronchodilator with Levalbuterol broncodilator

Yes, Salmeterol is long acting "sustainable", while Levalbuterol is short acting "Le" (less) "Val" (value)

Folks who have multiple sclerosis is degredation of the myelin, spinal cord injuries can used what med to help with muscle spasms?

bacflofen ----Muscle relaxant A client who has a spinal cord injury and takes baclofen can experience a decrease in the frequency and severity of muscle spasms and in flexor and extensor spasticity.

Selective beta blockers target beta 1 receptors which are cardio selective

beta blockers are beat blockers, slow HR treating hypertension!! beta-1 receptors r found in the heart and juxtaglomerular cells ex: Atenolol (A's r selective), Esmolol, METOPROLOL (I'm selective) A/E- *If med works to well and heart beat is extremely slow, blood can back up into lungs and peripheries

A nurse is teaching a client who has a new prescription for verapamil to control hypertension. Which of the following instructions should the nurse include?

increase amount of fiber in diet---Constipation is a common side effect of verapamil treatment. Increasing fluid and fiber intake can ease constipation.

short acting beta 2 agonists bronchodilators?

levalbuterol is short acting "Le" (less) "Val" (value)-----Albuterol

Glulisine (Apidra)

rapid acting insulin onset 15min-20min---eat breakfast immediately following admin---20-1 "peak"-5

What does St. Johns Wort "anti-depressant" do to most other drugs if taken with it?

reduces other drug effectiveness

meds given for stroke in pregnant woman

• ASA "aspirin" has shown to be safe during pregnancy of patient is at risk for stroke!..........No history of stroke .....high-risk women can take ASA "aspirin" 81mg daily History of stroke, multiple regimens are available; all includeHeparin • We can administer thrombolytic (tPA) to pregnant women who have had a thrombotic (blood clot) stroke because it Does NOT cross the placenta

Metoclopramide

Antiemetic and inc. gastric motility It can treat gastroesophageal reflux disease (GERD). It can also treat gastroparesis in patients with diabetes. *Intestinal obstruction is contraindicated

Nurse is planning discharge teaching for pt who has prescription for furosemide. Nurse should plan to include which of following statements in teaching?

"Drink a glass of milk with each dose of medication."The client should take furosemide with food or milk to reduce gastric irritation.

Nurse is providing teaching to pt who has UTI and new prescriptions for phenazopyridine and ciprofloxacin. Which of following statements by pt indicates need for further teaching?

"I should notify my provider immediately if my urine turns an orange color." Phenazopyridine is a urinary tract analgesic used to relieve pain and burning during urination. The medication can cause the client's urine to turn a reddish-orange color. This coloration is an expected effect of this medication, although it can stain clothing, and does not need to be reported to the provider.

Nurse is providing teaching to pt who has depression and new prescription for fluoxetine. Which of following statements by pt indicates understanding of teaching?

"I should take acetaminophen instead of ibuprofen for my headaches while taking this medication."------Fluoxetine (SSRI serotonin- reuptake inhibitor "too much serotonin" TONING u down too much)) suppresses platelet aggregation, which increases the risk of bleeding when used concurrently with NSAIDs and anticoagulants>>>>>Therefore, clients who are taking fluoxetine should take acetaminophen for headaches or pain, since acetaminophen does not suppress platelet aggregation. Concurrent use of St. John's Wort and fluoxetine "SSRI antidepressant" can increase the client's risk for serotonin syndrome, a potentially life-threatening complication. Manifestations of serotonin syndrome include confusion, hallucinations, hyperreflexia, excessive sweating, and fever.

A nurse is providing discharge instructions to a client who has a new prescription for amitriptyline to treat depression. The nurse should identify that which of the following client statements indicates an understanding of the teaching?

"I should watch for common reactions like dry mouth and constipation."Rationale: The nurse should reinforce that increasing dietary fiber, fluid intake, and chewing sugar-free gum can alleviate the anticholinergic effects of dry mouth and constipation

Nurse is providing teaching to pt about to use ethinyl estradiol/norelgestromin. Nurse should identify that which of following statements by pt indicates understanding of teaching?

"I will fold the sticky sides of the old patch together before disposing it.------"The client should fold the sticky sides of the old patch together and then place it in a childproof container to ensure safe disposal of the patch. The client should apply the patch within 7 days of menses to prevent ovulation and the need for another contraceptive method. The client should remove and dispose the old patch before applying a new patch to prevent toxicity by combining the remaining medication on the old patch with the medication on the new patch. The client should apply the patch once a week for 3 weeks and then go without the patch for 1 week to promote menstruation.

Immunosuppressant drugs

*Immunosuppressive drug---most cause bone marrow supression> low platelets "bleed risk" *hydroxychloroquine, methotrexate -----treats autoimmune diseases such as lupus, rheumatoid arthritis, psoriasis *CYCLOSPORIN, azathioprine ----given to prevent organ transplant rejection, life long drugs to lower immune response from rejecting organ; also given for autoimmune disease like rheumatoid arthritis or Irritable bowel disease

ACE inhibitors

"PRIL" Captopril, Enalapril, Afosiopril ----vasodilator "April has all A's walking across the vasodilated red Karpet (drug inc. potassium) who has to pee (low aldosterone secretions, Sodium and fluid loss) and her heart is racing "tachycardia" due to possible hypotension, she has a persistent dry cough and just found out she was diagnosed with pancytopenia.......she thinks the low neutrophils are causing her cough but its actually the substance P" Antihypertensive. Blocks ACE in lungs from converting angiotensin I to angiotensin II (powerful vasoconstrictor) causing VASODILATION Decreases BP, Decreased Aldosterone secretions, Sodium and fluid loss "vasodilates"--- can cause hyperkalemia ACE inhibitors affect the respiratory system is thought to be through an increase of substance P,this will increase the cough reflex *can cause Orthostatic Hypotension---Check BP before giving (hypotension) *neutropenia is an A/E of Captopril Adverse effects---- of ACE inhibitors include angioedema, persistent dry cough, altered taste, fatigue, headache, hyperkalemia, hypotension, photosensitivity, proteinuria, rash, tachycardia, and pancytopenia.

A nurse is providing discharge teaching to a client who has angina pectoris and a new prescription for verapamil. The client tells the nurse, "My brother takes verapamil for high blood pressure. Do you think the provider made a mistake?" Which of the following responses should the nurse make?

"Verapamil is used to treat both high blood pressure and angina." "think someone on the stage rapping and artist has his calcium channel blockers protecting him.... he is moving slow to prevent orthostatic hypotension....his song is about foods high in fiber so you do not get constipated.....the calcium channel blockers are fed up of the nifedipine/amlodipine phenes"

cyclosporine

"cyclosporin" ----immunosurpressive drug for organ transplants Drug class: Calcineurin inhibito--- not an antibiotic like cephalosporin!!! most common indication: given to prevent organ transplant rejection, life long drugs to lower immune response from rejecting organ also given for autoimmune disease rheumatoid arthritis, lupus, psoriasis, IBD "irritable bowel disease" A/E- 1. bone marrow supression---before med. admin check WBC (5,000-10,000) "notify provider for signs of infection", "Avoid Crowds", "No live vaccines like herpes booster + Shingles", "Use soft bristle tooth brush" Platelets (140,000-400,000) "monitor for bleeding" 2. Do not give to pregnant patients---use contraceptives 3. Most common A/E-- do not report to provider is GINGERVAL HYPERPLASIA 4. always avoid grapefruit juice

Dantrolene (Dantrium)

-skeletal muscle relaxant -Antidote for malignant hyperthermia (inc. hypermetabolic activity) rare problem potentially triggered by genes or Succinylcholine (another type of muscle relaxant that has been used for rapid sequence induction and for procedures requiring only a brief duration of muscle relaxation or general anesthesia) Dantrolene (Dantrium) muscle relaxer that is used to treat muscle spasticity (stiffness and spasms) caused by conditions such as a spinal cord injury, stroke, cerebral palsy, or multiple sclerosis (degeneration of the myelin sheath) contraindication- those who have cirrosis "liver disease" cannot take this med. since it is hepatotoxic---for this reason liver function tests are monitored throughout treatment

Lithium toxicity

0.5-1.5 Imbalance of Na can cause lithium toxicity---lithium binds to Na channels---if there is less Na binded then lithium can bind more----be aware of fluid imbalances due to vomiting/ diarrhea/ diuretics which deplete Na and water Avoid NSAID inc. li levels A/E--Blurred vision, tinitis, DRY MOUTH* (patient should not expect to have these A/E)

Clozapine "schizophrenia" ---antipsychotic

"think of Schizophrenic person "ChlorPROMaZine" at prom with Amantadine "Amanda" as his date who is his antidote and decreases his extrapyradimal effects; Chlorpramazine has a twin who also has extrapyradimal effects named Clozapine who also has Schizophrenia and likes to hide in the cloZet..... he is severely sick (agranulocytosis, anticholinergic effects, high cholesterol, diabetes mellitus, Myocarditis (has a big heart), Oculogyric crisis "rolling back of the eyes", Hormonal side effects)" § risk for agranulocytosis § anticholinergic effects §Labs- Cholesterol levels ---cause hyperlipidemia; Induced Diabetes>>>>inc. cortisol due to constant SNS "anticholinergic" effect § MYOCARDITIS occurring 30 ays after starting medication--S/S of Myocarditis: chest pain, palpitations, tachycardia, arrythmias, dyspnea, tachypnea "inc.RR", fever, peripheral edema, unexplained fatigue---if occurs avoid in future § Extrapyramidal side effects— • Pseudoparkinsonism- trmmor, shuffling gait, drooling -more common in women and elderly that r dehydrated • Bradykinesia---slow movement • Akasthisia-restlessness • Dystonia (treatment: anticholinergics or diphenhydramine) ---involuntary muscle spasm in face "laryngospasms", arms, leg, and neck---life threatening!! • Oculogyric crisis---rolling back of the eyes "medical emergency" • Tardive dyskinesia—chronic long term use—permanent—stiff neck, tonged movement, § Hormonal side effects-typical antipsychotics • Decreased libido • Erectile dysfunction • Retrograde ejaculation • Gynecomastia---male clint with this may think external forces are taking over his body and turning him into a woman!!! • Amenorrhea---psychopathic woman may think she is pregnant • Educate mental patients on these crazy side effects as not to confuse them with their mental condition!!!

A nurse is reviewing a new prescription for oxcarbazepine with a female client who has partial seizures. Which of the following instructions should the nurse include?" Select all that apply

"use caution if given a prescription for a diuretic med?", "consider using an alternate form of contraception if you are using oral contraceptions", "avoid driving until you see how the med affects you", "notify your provider if you develop a skin rash"

List of drugs for alcohol The FDA has approved three oral medications (disulfiram, acamprosate, and naltrexone) and one injectable medication (extended-release injectable naltrexone) for the treatment of alcohol dependence or the prevention of relapse to 7,24 history of medication adherence. Although further research with large patient samples is required before definitive advice can be offered on which medication to select for a particular patient, information for matching patients to particular pharmacotherapies is Mechanism of Action. The disulfiram reaction is caused by a blockade of aldehyde dehydrogenase, which causes an accumulation of acetaldehyde when alcohol is ingested. When this occurs, the physical reaction can include 40 Formulation. Disulfiram is manufactured as a white to off-white odorless and almost tasteless powder. It is supplied in 250 mg and 500 mg alcohol use.

(disulfiram, acamprosate, and naltrexone) *chlordiazepoxide-----1st line medication for alcohol withdrawal symptoms, and tremor..... sedative

A nurse is providing teaching to a client who has hypertension and a new prescription for captopril. Which of the following instructions should the nurse provide? A.) Do not use salt substitutes while taking this medication B.) Take the medication with food C.) Count your pulse rate before taking the medication D.) Expect to gain weight while taking this medication

** A.) Do not use salt substitutes while taking this medication-------The client should avoid salt substitutes, as most of them are high in potassium (remember it decreases Aldosterone "inc. na reabsorb" secretions causing Sodium and fluid loss and "vasodilates"--- can cause hyperkalemia) "April has all A's walking across the vasodilated red Karpet (drug inc. potassium) who has to pee (low aldosterone secretions, Sodium and fluid loss) and her heart is racing "tachycardia" due to possible hypotension, she has a persistent dry cough and just found out she was diagnosed with pancytopenia.......she thinks the low neutrophils are causing her cough but its actually the substance P"

Herb gingko biloba

******decreases platelet aggregation---inc. risk for bleeds**** *good for clients with deep vein thrombosis

two meds used to treat diabetes insipidus?

*1st line drug------desmopressin= ADH *hydrochlorothiazide= reduces urine production 30%-50% its in the distal convoluted tubule; not as affective as loop diuretics ----thiazides have been used in patients with nephrogenic diabetes insipidus (NDI) to decrease urine volume, but the mechanism by which it produces the paradoxic antidiuretic effect remains unclear.

What does terbutaline do for women in pre-term labor?

*Activates SNS----Activates Beta 1 *(Heart- tachycardia) and Beta 2 (lungs/ broncoconstriction) Terbutaline is approved to prevent and treat bronchospasm (narrowing of airways) associated with asthma, bronchitis, and emphysema. The drug is sometimes used off-label (an unapproved use) for acute obstetric uses, including treating preterm labor and treating uterine hyperstimulation.

Valproic Acid

*Anticonvulsant *It can treat seizures and bipolar disorder. *It can also help prevent migraine headaches.

Erythromycin

*Give on empty stomach (antibiotics) -report persistent diarrhea to the provider---can cause super infection b/c it kills good bacteria in the GI tract* -Antacids don't reduce rate of absorption for this med..... it does for azithromycin -may cause tennitis---hearing loss

Heparin (ENOXAparin) is administered to a client who has deep vein thrombosis, how should nurse administer drug?

*Heparin "ENOXAprin" is for the Hospital in abdomen or sub q, *apply firm pressure for 5 sec, *22 G needle to BIG--- use a 25G needle this will decrease bleed risk; can use electric pump This drug is NOT thrombolytic "clot buster" but STOPS clot formation "anticoagulant" and enlargement Nurse should monitor the aPTT levels (60-80 sec) *(Warfarin "Coumadin"= K given at the same time but is maintained at home)

Hydroxychloroquine (Plaquenil)

*Immunosuppressive drug---- treat lupus and arthritis; Anti-parasite----It can treat and prevent malaria. Hydroxychloroquine has been studied for the treatment and prevention of coronavirus disease 2019 "(COVID-19)"; Hydroxychloroquine and chloroquine are FDA-approved to treat or prevent "malaria"; Hydroxychloroquine is also FDA-approved to treat "autoimmune disease" A/E-eye damaging

Anti-inflammatories: decreases swelling and mucus production...used as long-term treatment to control asthma not an acute attack. Anti-inflammatories used to treat asthma include?

*Inhaled corticosteroids: "FluticaSONE", "Budesonide", "BeclomethaSONE" -watch for thrush (use spacer with inhaler and rinsing mouth after administration) -lowers immune system - increases osteoclast activity---risk for osteoporosis and cataracts (cloudy lens on the eyes)May be given IV or PO for severe asthma attack. -increases glucose levels in the blood -*oral steroids can suppress growth in children "adrenal supression" that's why inhaled route attacking the source is standard* *Leukotriene Modifiers (oral): "Montelukast"+ "Zileuton" blocks the function of leukotriene which causes the smooth muscle on the airways to constrict + plays a role in mucus production. When this function is blocked it leads to the relaxation of the smooth muscle and decreased mucous production...NOT for an acute attack *Immunomodulator (subq): "Omalizumab" blocks the role of the immunoglobulin IgE, which will decrease the allergic response...hence asthma attacks given subcutaneously used when patient's asthma is poorly controlled and other treatments are not working NOT used as a quick relief NO LIVE vaccines while receiving *Nonsteroidal Anti-Allergy: "Cromolyn" (inhaled) long term use....not for quick relief in emergency situations stops mast cells from secreting histamine. This decreases inflammation. While receive the patient may temporarily experience sneezing, burning in nose, itchy/watery eyes, bad taste in mouth.

Naltrexone (ReVia)

*decreases the CRAVINGS for alcohol---gives person will power to not pick up multiple drinks ---- drug not priority during ACUTE withdrawal ----chlordiazepoxide is! Patient can take the drug before they drink to decrease the positive effects of alcohol giving them more of a possibility to now pick up an additional drink *treat both opioid and alcohol use disorders.

effects of low estrogen in the body?

*regulates glucose and lipid metabolism---weight gain may be evident *low estrogen may cause spotting between periods----note: spotting can occur with high or low levels of estrogen----- here is why? "LOW LEVELS of estrogen result in intermittent spotting that may be prolonged but is usually light in the amount of flow/ HIGH LEVELS of estrogen for prolonged periods of time result in lengthy periods of amenorrhea followed by acute, often heavy, bleeding with excessive blood loss. ESTROGEN BREAKTHROUGH bleeding occurs when excess estrogen stimulates the endometrium to proliferate in an undifferentiated manner. With INSUFFICIENT PROGESTERONE to provide structural support, portions of the endometrial lining slough at irregular intervals. (high levels of estrogen also can cause headache, breast tenderness, and nausea" *combination oral contraception may cause low estrogen---- this leads to early menopause

Clozapine (Clozaril)

- Atypical Antipsychotic medication "Schizophrenia" - A/E----MYOCARDITIS occurring 30 ays after starting medication--S/S of Myocarditis: chest pain, palpitations, tachycardia, arrythmias, dyspnea, tachypnea "inc.RR", fever, peripheral edema, unexplained fatigue---if occurs avoid in future - May cause severe AGRANULOCYTOSIS (weekly blood draws/ CHECK FEVER) low WBC, Platelets--- this causes LETHARGY and MUSCLE PAIN -A/E--anticholinergic effects--- urinary retention (no need to record urinary output), constipation, dry mouth blurred vision--promote sugar less candy and frequent oral care

Rivastigmine

-Anticholinesterase ----works by preventing the breakdown of a certain natural substance (acetylCHOLINE) in your body. For Alzheimer disease

A nurse is providing discharge teaching about handling medication to a client who is to continue taking oral transmucosal fentanyl raspberry - flavored lozenges on a stick. Which of the following information should the nurse include in the teaching?

-Store unused medication sticks in a storage container -move stick in different locations of the mouth, DO NOT leave in one location, for best absorption -fast effect 10-15min!!--Not an hour

Hepatitis A vaccine

-child may loose appetite after receiving the vaccine, tell parent that this may last a few days *Newborns born to infected mothers should receive the hep B vaccine and hep B immune globulin within 12 hours of birth *A+E are fecal oral the rest are blood born

oral contraceptives

-decrease folate, B6, C -You can miss up t6o 7 pills with little risk in getting pregnant as long as you have taken the pills regularly for the previous 3 weeks -oral contraceptive containing estrogen inc. risk of deep vein thrombosis, inc. risk of breast cancer, promote mineral deposition, benefits cholesterol levels by reduces LDL + inc. HDL

a nurse is teaching a client about raloxifene. which of the following information should the nurse include?

-increase physical activity by taking walks.....risk for dvt -use a contraceptive if there is any possibility of pregnancy----drug is a category x----remember X in raloXifene -increase intake of calcium and vitamin D to promote calcium absorption in menopausal women -medication can exacerbate hot flashes "Think of This of lady named RaloXifene who has s/s of low estrogen (hot flashes, decreased libido, vaginal dryness) but has inc. bone density and inc. risk for clots (DVT/ leg cramps) as if she had high amounts of estrogen......... The X signifies she can't have babies!"

Steriods types: prednisone, dexamethasone, hydrocortisone, fludrocortisone

-makes the liver more resistant to insulin causing HYPERGLYCEMIA -decreases WBC---fever is priority -Inc. Na and water absorption.....patient should check daily weights 2Ibs or more a day is bad! -Corticosteroids tend to both reduce the body's ability to absorb calcium and increase how fast bone is broken down. -those in surgery must have an increased dose due to increase stress -slowly taper off -patient has a sore on their leg that won't heal due to low immune system - used for lupus and it lowers inflammatory response

acetylcysteine>>>> mucolytics (hypertonic saline solution, acetylcysteine)

1. Treats- Acetominophen overdose---given IV/Oral 2. Treats- cyclic fibrosis/ chronic bronchitis my loosening secretions b/c of hypertonic state--inhaled---patient must cough up sputum before treatment to med can reach bronchioles this drug thins the mucous membrane out and causes secretions to break loose and come out through cough----helps those w cystic fibrosis/ chronic bronchitis---INHALED can cause bronchospasms , be careful w/ asthma patients A/E- can irritate the airways while thinning out mucus secretions and producing a cough---- causing BRONCHOSPASMS

what 3 common A/E occur with atypical anti-psychotic med. clozapine?

1. insulin resistance----diabetes 2. dyslipidemia---check triglyceride + cholesterol levels 3. "think of Schizophrenic person "ChlorPROMaZine" at prom with Amantadine "Amanda" as his date who is his antidote and decreases his extrapyradimal effects; Chlorpramazine has a twin who also has extrapyradimal effects named Clozapine who also has Schizophrenia and likes to hide in the cloZet..... he is severely sick (agranulocytosis, anticholinergic effects, high cholesterol, diabetes mellitus, Myocarditis (has a big heart), Oculogyric crisis "rolling back of the eyes", Hormonal side effects)"

Herb: Garlic

1. lowers cholesterol level 2. ******decrease platelet aggregation--- affects clotting + interacts w/ other meds

Penicillin admin. for pneumonia A/E

1. pain at the injection site 2. prolonged motor dysfunction 3. laryngeal edema (Evaluate 1st) 4. Temp 99.7F

A nurse on an inpatient unit is caring for a client who has schizophrenia and recently started taking risperidone. Which of the following actions should the nurse take? a. Implement fall precautions for the client. b. Monitor the client's thyroid function c. Place the client on a fluid restriction d. Discontinue the medication if hallucinations occu

A a. Implement fall precautions for the client.i. Risperidone can cause orthostatic hypotension and dizziness, which can lead to falls. Therefore, the nurse should initiate fall precautions for the client. b. Monitor the client's thyroid functioni. Risperidone does not affect thyroid function. The nurse should monitor the client's CBC for anemia, thrombocytopenia, leukocytosis, and leukopenia as well as the client's liver function for elevated AST or ALT levels. c. Place the client on a fluid restrictioni. Risperidone can cause constipation, diarrhea, or dry mouth. Therefore, the nurse should encourage increased intake of fluids. d. Discontinue the medication if hallucinations occur. i. Hallucinations are an expected manifestation of schizophrenia. Risperidone is an antipsychotic medication used to treat manifestations of schizophrenia, including positive symptoms such as hallucinations. Therefore, the nurse should not discontinue the medication.

calcium channel blocker used to treat (pectoris angina/hypertension)

Amlodipine Nifedipine "when you see PINE this is usually a calcium channel blocker" ----can cause gingival hyperplasia

Meds for asthma

AIM A- Beta- 2 agonist----Albuterol I- Anticholinergics-- Ipratropium (form of atropine)---cant pee but bhronchodilates---treats COPD M- Methylxanthines-- Theophylline--- "Theo is a God + will see God if taking this med due to toxic effects" toxic + cause tachycardia, do not mix w/ caffeine (10-20 range); Methylxanthines (Aminophylline/ bronchodilator----; Methyl-preniso-lone (Solu Medrol) sound like prednisone which is how u know it's a steroid....used last b/c slow acting; Methylxanthines-- Theophylline--- "Theo is a God + will see God if taking this med due to toxic effects" toxic + cause tachycardia, do not mix w/ caffeine (10-20 range) SLAM S- Steriods--Beclomethasone (surpasses the immune system= sores in mouth= slow wound healing----- wash teeth q. use to avoid thrush, wash device with tap water 2x/week or daily---wash weekly for albuterol, can cause sepsis + sickness, sugars may be increased which is normal b/c cortisol "steroids" inc. glucose levels, *oral steroids can suppress growth in children "adrenal supression" that's why inhaled route attacking the source is standard*) L- Leukotriene inhibitors---Monteleukast--kast out leukotrienes which cause inflammation therefor reduces mucus production + opens bronchioles+ not for acute asthma attacks; 3 L's (Luke likes to sing "opens up airways"/ Long term management/ Long onset "takes 1-2 weeks to be effective"---- *patient states they have been on moteleukast for 5 days and med is not working! Nurse must advice patient to wait 1-2 weeks A- Albuterol M- Mast cell stabilizers "Cromolyn"---long term us--histamine stabilizer

Isotretinoin (Accutane)

Acne medication NEED TWO negative pregnancy test before starting medication --- has been linked to depression and suicidal ideation (rare) Educate: use two forms of brith control Can NOT donate blood for at least one month to ensure preggos do not receive donation Pregnancy X category

Alendronate versus risedronate "onate"

Alendronate versus risedronate - *Alendronate and risedronate have been compared in one randomized trial and in some retrospective observational trials. In the randomized trial, alendronate increased bone density more than risedronate at all sites after 12 months *Alendronate has been shown to be more effective than risedronate at increasing BMD and reducing bone turnover without increasing the risk for side effects.

Pegloticase vs Allopurinol

Allopurinol is the 1st line of treatment for CHRONIC gout----long term treatment. "think PeglotiCASE vs ALLopurinol.....you ALL got a Case with Chronic gout" Pegloticase is prescribed if treatment with allopurinol is ineffective---It is a 3RD line treatment in those in whom other treatments are not tolerated---This drug is administered by infusion intravenously.----*A/E= patient is expected to have inc. gout manifestations, client can take NSAID such as naproxen to reduce manifestations

Antidote for Chlorpromazine used to treat schizophrenia?

Amantadine ----- "think of Schizophrenic person at prom with Amantadine "Amanda" for their date as their antidote Antiviral drug and Dopamine promoter It can treat and prevent the flu (influenza type A). It can also treat Parkinson's disease and parkinson-like symptoms caused by certain medications.

Calcium Channel Blockers

Amlodipine Diltiazem Nifedipine Verapamil

Amphetamines

Amphetamine is a central nervous system STIMULANT that is used in the treatment of attention deficit hyperactivity disorder, narcolepsy, and obesity. Amphetamine was discovered in 1887 and exists as two enantiomers: levoamphetamine and dextroamphetamine. *Narcolepsy is a chronic sleep disorder characterized by overwhelming daytime drowsiness and sudden attacks of sleep.

Nurse is caring for pt who is receiving haloperidol. Nurse should identify which of following findings as adverse effect of med?

An adverse effect associated with haloperidol is the development of extrapyramidal manifestations such as dystonia, pseudoparkinsonism, and akathisia. Haloperidol, an antipsychotic neuroleptic medication, can cause CNS adverse effects such as seizures, confusion, and neuroleptic syndrome. However, paresthesia is not an adverse effect of haloperidol Haloperidol has anticholinergic properties that can cause sensory adverse effects such as increased intraocular pressure, blurred vision, and dry eyes.

Analgesia vs Anesthesia

Analgesia -> loss of ability to feel pain ONLY "pain meds"----may cause sedation as an A/E; does not interfere with sensation "temp, touch, taste" Anesthesia -> loss of ability to sense all sensations, including pain; can cause loss of consciousness

Acarbose

Anti-diabetic medication; Acarbose lowers your blood sugar by preventing the breakdown of starch into sugar. It may be used alone or in combination with another type of oral diabetes medicine called a sulfonylurea. It can improve glycemic control in adults with diabetes. *tell client to take medication with food 3 x a day *admin by mouth *can be used with insulin, metformin, or sulfonylurea *used along with diet and exercise

Sulfasalazine

Anti-inflammatory----It can treat inflammatory bowel disease- ulcerative colitis + crohn's disease; rheumatoid arthritis----reduces inflammation, pain and swelling in your joints A/E- 1. blood disorder--Bone marrow suppression causing agranulocytosis "low amount of WBC in body", hemolytic anemia "RBC's destroyed faster than they are replaced", macrocytic anemia "bone marrow produces large RBC's" ----clients blood count should be continually monitored and checked before administering the med! 2. Nausea 3. Fever 4. Rash 5. Arthralgia 6. Hepatotoxicity. Sulfasalazine, like other sulfonamides, causes a characteristic idiosyncratic liver injury that has features of drug-allergy or hypersensitivity (nurse should tell patient to report jaundice---yellow discoloration in skin + yellow/orange discoloration in urine)****** Contraindications- 1. those w/ sulfa allergy---including thiazide diuretic, salycilates, https://www.youtube.com/watch?v=KHxE-_pTdKM https://www.youtube.com/watch?v=ULkyURIvYgc&list=PLuoy1mRgIUpRCUTDWURlTtvbj34TkY3AX&index=2

Digoxin

Antiarrhythmic and Blood pressure support It can treat heart failure and heart rhythm problems. Digoxin (0.5-2) helps make the heart beat stronger and with a more regular rhythm. Hold if Heart Beat is less than 60 beats/min----sign of dig. toxicity *blurred vision is an A/E, notify provider----sign of dig. toxicity---- person will not experience weight gain as this is a right sided heart problem----dig. may cause fluid in lungs instead interact with calcium gluconate----binds to same site as calcium.. if we have too much calcium then dig will remain in the blood as calcium has taken up all the binding sites.... this will cause dig. toxicity.

Amiodarone (Cordarone)

Antiarrhythmic--- used to suppress ventricular and supra ventricular tachyarrhythmias "picture guy Amiodarone with blue face coughing (pulmonary toxicity) and is putting on sunscreen to decrease his photosensitivity"

ADENosine------not the same as atropine

Antiarrhythmic---is a chemical found in human cells; Adenosine blocks electrical signals in the heart that cause irregular heart rhythms. ----AdeNoSine........Ade No it's a Sin to stop the heart....he stop breathing for a sec also>>>stops the heart and then reboots Atrial Tachycardia (supra ventricular) + v.fib--- must slam drug *causes DYSPNEA due to BRONCHOCONSTRICTION--- transient drug has short half-life of 10-sec----- willing to take that breathing risk for 10 sec to stop WEIRD electric impulse!!!!

Linezolid (Zyvox)

Antibiotics It can treat bacterial infections, including skin infections and pneumonia, staph infection A/E must report to provider: >irreversible peripheral neuropathy >reversible optic neuropathy "EYES" Typical A/E- Naseau, headaches, insomnia

Ipratropium Bromide

Anticholinergic blocking acetylcholine (can't see, pee, spit, or shit)---- a medication which opens up the medium and large airways in the lungs. It is used to treat the symptoms of chronic obstructive pulmonary disease and asthma. Used 2nd in line for acute asthma attacks "AIM" A- Albuterol I- Ipratropium---(form of atropine longer acting than albuterol- COPD) M- Methylprednisolone (Solu Medrol)

Tiotropium Bromide (inhalation)

Anticholinergic blocking acetylcholine (can't see, pee, spit, or shit)--a long-acting, antimuscarinic "bronchodilator" used in the management of chronic obstructive pulmonary disease (COPD) and asthma. * NO swallowing tiotropium capsules---put capsule into inhaler device and inhale Used 2nd in line for acute asthma attacks "AIM" A- Albuterol I- Ipra"tropium" M- Methylprednisolone (Solu Medrol)

Amitriptyline (Elavil)

Antidepressant and Nerve pain medication! Ami be tripping cause she be so depressed causeshe is experiencing all these anticholinergic effects Blocks acetylcholine....blocking PSNS-- SNS-----The nurse should expect the client to have a dry mouth due to the blocking of acetylcholine receptors that cause anticholinergic responses........ tricyclic antidepressant!!!! Ami trips on things cause she is so depressed, ----get tachycardia while falling---may cause ECG changes---she trips on things in the deserts shows it has drying effects blurred Vision (dry eyes)--anticholinergic effects

sodium polystyrene sulfonate (Kayexalate)

Antidote for hyperkalemia----KayEXalate helps K EXIT the body! *Helps the large intestine remove excess K *No contrast is used during admin----Given PO or Enema *Not used for bedridden clients who are constipated---ensure close assess to bathroom for loose stools and assist to the bathroom *Does not decrease bacteria within the intestine, Neomycin enema do this! *Must "assess the abdomen" ensure normal bowel function and frequency of stools before med. admin to prevent intestinal necrosis *Drink fluids after med. admin to promote effective of lactulose *3.5-5.3 K levels is indicative of med. effectiveness (creatinine, phosphate, or calcium are not evaluated for the effectiveness for the drug) *Look for signs of hypokalemia (k help muscle pump/ low K= muscle weakness, low muscle pumps "palpitations" + flat T waves in heart, lethargy and cramping in muscles *Assess for bowel function before looking for signs of hypokalemia

Famotidine (Pepcid) other: Cimetidine, rimitidine

Antihistamine and Antacid (heartburn relief); H2 blocker "takes 2 Humans to dine" ---duodenal ulcer; less effective w/ smokers Note: PPI for gastric ulcer *med is LESS effective in people who SMOKE--patient should quit smoking or avoid smoking after last med. admin *CAN take on an empty stomach or with food--- food does not affect absorption of drug *take anti-acid 30-60min after taking famotidine not at same time---You can take an antacid (such as TUMS, Rolaids, Maalox, Mylanta, and Milk of Magnesia) with famotidine (Pepcid AC) for quicker heartburn relief. However, you should consult with your healthcare provider first before taking the those medications together. *H-2 blocker is hepatotoxic and can cause JAUNDICE Contact provider is you experience----A/E- dizziness*, Nausea, bloody stools (blood dyscrasias "disease" causing thrombocytopenia*), drowsiness, headache contact provider, jaundice "yellowing of the skin or eyes"

Nurse is reviewing medical record of pt who has HTN. Nurse should identify which of following findings as contraindication for receiving propranolol?

Asthma is a contraindication for receiving propranolo------Beta Blockers affect the Bronchioles "airway"! Propranolol is an adrenergic antagonist which blocks the beta2 receptors in the lungs, causing bronchoconstriction and leading to serious airway resistance and possibly respiratory arrest.

A nurse is an emergency department is caring for a client who has myasthenia gravis and is in a cholinergic crisis. Which of the following medications should the nurse plan to administer?

Atropine "ANTICHOLINGERGIC/SNS" (reverse cholinergic toxicity)---ALSO used to open bronchioles myasthenia gravis have muscle weakness PNS inc. muscle activity so cholinergics are given......"dry asthenia gravis)

Atropine

Atropine "anticholinergic/ SNS"----a prescription medicine which can inc. HR, cause pupil dilation, reduce salvation, reduce bronchiole secretion, inc. bronchodilation, may cause muscle weakness of urinary and ciliary "eye" muscle *an antidote for overdose of cholinergic "PNS" drugs (Pyridostigmine "AcetylCHOLINE" is used to improve muscle strength in patients with a certain muscle disease like myasthenia gravis or mushroom poisoning) *patient can experience cyscloplegia "blurred vision" (paralysis of the ciliary muscle preventing ability to focus far or near) after administration of atropine eye drops---inhibits PNS (stops rest + digest which includes muscle activity) treat the symptoms of: 1. low heart rate(bradycardia/ heart block) 2. Anticholinergic affects----reduce salivation and bronchial secretions before surgery 3. antidote for overdose of cholinergic drugs or mushroom poisoning.

A nurse is caring for a client who has a new prescription for clonidine. The nurse should inform the client that which of the following findings is an adverse effect of this medication? a. Diarrhea b. Dry mouth c. Photophobia d. Bruising

B b. Dry mouthi. Clonidine is an indirect-acting antiadrenergic agent used for hypertension, severe pain, and attention deficit disorder. The nurse should inform the client that dry mouth, or xerostomia, is a common adverse effect of clonidine a. Diarrheai. Constipation is a common adverse effect of clonidine c. Photophobiai. Photophobia is not an adverse effect of clonidine. However, the medication can dry eyes d. Bruisingi. Bruising is not an adverse effect of clonidine, although the medication can cause other skin manifestations, such as rashes, transdermal clonidine patches can cause localized skin reactions

Acamprosate (Campral)

Maintains Abstinence from Alcohol---works in the brain by decreasing cravings and urges to use alcohol. This allows people who take the medication to control urges to drink and help to continue to not use alcohol.

A nurse is caring for a client who is receiving heparin therapy via continuous IV infusion to treat a pulmonary embolism. Which of the following findings should the nurse identify as an adverse effect of the medication and report to the provider?

Blood in the urine (heparin toxicity)------also bruising, hematoma, hypotension, and tachycardia may occur *A hematoma is a bad bruise. It happens when an injury causes blood to collect and pool under the skin. There are also subdural hematoma

Nurse is assessing pt who has received atropine eye drops during eye examination. Which of following findings should nurse expect as adverse effect of med?

Blurred vision---Blurred vision is an expected finding following the administration of atropine eye drops. This is due to the cycloplegic effects of the medication, which cause distant objects to appear blurry to the client. Dilation of pupils, or mydriasis, is an expected finding following the administration of atropine eye drops. A client who has received atropine eye drops can experience photosensitivity, which causes difficulty seeing in brightly lit areas due to the muscarinic receptors causing mydriasis.

A nurse is caring for a school-age child who is taking valproic acid. The nurse should expect the provider to order which of the following diagnostic tests? a. Chest x-ray b. ABGs c. Serum liver enzyme levels d. Urine culture and sensitivity

C a. Chest x-rayi. It is not necessary to assess the lungs of a client who is taking valproic acid b. ABGs i. It is not necessary to assess the ABGs of a client who is taking valproic acidc. Serum liver enzyme levelsi. Valproic acid can cause hepatic toxicity. Therefore, the nurse should expect the provider to prescribe laboratory tests to assess the child's liver function prior to and periodically during therapyd. Urine culture and sensitivityi. It is not necessary for a client who is taking valproic acid to undergo a urine culture and sensitivity

A nurse is teaching a client who has a new prescription for escitalopram for treatment of generalized anxiety disorder. Which of the following statements by the client indicates understanding of the teaching? A- i should take the medication on an empty stomach B- i will follow a low-sodium diet while taking this med C- i need to discontinue this med slowly? D- i should not crush this med before swallowing

C- when discontinuing escitalopram, the client should taper the medication slowly according to a prescribed tapered dosing schedule to reduce the risk of withdrawal syndrome Selective serotonin reuptake inhibitors (SSRIs)

A nurse educator is reviewing medication metabolism at an in-service presentation. Which of the following factors should the educator include as a reason to administer lower medication dosages? (Select all that apply.) A. Increased renal excretion B. Increased medication-metabolizing enzymes C. Liver failure D. Peripheral vascular disease E. Concurrent use of medication the same pathway metabolizes

C---liver is metabolizing med. slower inc. the amount in the system----can be toxic E--- NOT B----anything that increases the rate of metabolism (e.g., enzyme induction) of a pharmacologically active metabolite will decrease the duration and intensity of the drug action.

A nurse in an acute care facility is caring for a client who is receiving IV nitroprusside for hypertensive crisis, The nurse should monitor the client for which of the following adverse reactions to this medication? A. Intestinal ileus B. Neutropenia C. Delirium D. Hyperthermia

C. Delirium----symptoms caused by cyanide intoxication are non-specific and include metabolic acidosis, headache, seizures, coma, delirium, weakness, dyspnea Nitroprusside is rapidly hydrolyzed (half-life, 11 minutes) and releases free cyanide,

A nurse is caring for ancient who is receiving moderate sedation with diazepam IV. The client is overstated. Which of the following medications should the nurse anticipate administering to this client?a.Ketamine B.Naltrexone C.Flumazenil D.Fluvoxamine

C.Flumazenil

A nurse is providing teaching for a client who is withdrawing from alcohol and has a new prescription for propranolol. Which of the following information should the nurse include in the teaching? A. increases the risk for seizure activity B. Provides a form of aversion therapy C. Decreases Cravings D. Results in mild HTN

C. propranolol is an adjunct medication used during withdrawal to decrease the client's craving for alcohol Article: Effects of beta-blocking drugs in alcohol withdrawal:-----an increase in adrenergic activity may also play a part in alcohol withdrawal symptoms, suggesting a potential efficacy of beta-blocking drugs.

Tiotropium is an anticholinergic used for?

COPD---- stops PSN, which stops secretions!! Never swallow pill-- insert in inhaler device and then inhale Contraindicated in: 1. Glaucoma patients "already can't see"/ 2. folks with peeing issues "urinary retension, BPH "benign prostate hypertrophy" 3. Poop + spit issues= bowel obstructions

Celecoxib (Celebrex)

COX-2 inhibitors are a subclass of nonsteroidal antiinflammatory drugs (NSAIDs)----can be used to treat ACUTE Rheumatoid arthritis* Rheumatoid arthritis that is chronic give steroid like prednisone NSAIDs work by reducing the production of prostaglandins, chemicals that promote inflammation, pain, and fever.

A nurse is teaching a client who is taking levodopa/carbidopa to treat Parkinson's disease. Which of the following nursing interventions should the nurse include?

Change position slowly to prevent orthostatic hypotension***Levodopa/carbidopa can cause orthostatic hypotension.

Vincristine

Chemotherapy-----It can treat leukemia "lymphocytic leukemia" and many other types of cancer, usually in combination with other medications. A/E: *Paresthesia *Alopecia *Stomatitis *constipation

Nurse is providing teaching to pt who is to start taking sumatriptan. Which of following adverse effects should nurse instruct pt to monitor for and report to Dr?

Chest pressureSumatriptan is an antimigraine agent which can cause coronary vasospasms, resulting in ANGINE. The client should report chest pressure or heavy arms to the provider.

Colesevelam (Welchol)

Cholesterol medication----It can lower cholesterol levels ---causes the liver to make bile acid (made of cholesterol), which is than removed from the body.

Neostigmine (Prostigmin)

Cholinesterase inhibitor----choline break-down inhibitor. Prevents break down of acetylcholine.....inc. choline duration resulting in improved transmission of nerve impulses. *Used for myasthenia gravis (muscle weakness)* Precautions/interactions: do not administer if systolic BP is less than 90 mm Hg (120/80)----as in inc. PSNS "rest + digest" Side effects: slow HR, chest pain, weak pulse, increased sweating and dizziness, feeling like need to pass out, weak or shallow breathing, urinating more than usual, seizures, and trouble swallowing. Wear medic alert bracelet. Monitor for cholinergic crisis.

Treatment for duodenal ulcer?

Cimetidine Antihistamine and Antacid It can reduce acid in the stomach to treat ulcers and acid reflux. Heartburn Relief *emesis that looks like coffee ground is indicative of GI bleed due to med not working report to provider *This med may cause antiandrogenic effects like erectile dysfunction (will reverse when med. is discontinued), myalgia or muscle pain (common no need to report), enlarged breast tissue in males/gynecomastia (will reverse when med. is discontinued),

Nurse is planning to teach about inhalant meds to pt who has new Dx of exercise-indused asthma. Which of following weds should nurse plan to instruct pt to use prior to physical activity?

CromolynCromolyn sodium stabilizes mast cells, which inhibit the release of histamine and other inflammatory mediators. The client should use cromolyn 10 to 15 min before planning to exercise to prevent bronchospasms.

A nurse is teaching a client who is taking digoxin and has a new prescription for colesevelam. Which of the following instructions should the nurse include in the teaching? A. "Take digoxin with your morning dose of colesevelam." B. "Your sodium and potassium levels will be monitored periodically while taking colesevelam." C. "Watch for bleeding or bruising while taking colesevelam." D. "Take colesevelam with food and at least one glass of water."

D. "Take colesevelam with food and at least one glass of water."

Alosetron (Lotronex)

Diarrhea medication-----It can treat irritable bowel syndrome (IBS)---AAHHH my stool is loose, I need Alosetron!!! A/E- constipation if med works too well, provider may reduce dose *Will not increase weekly but give beginning of the month and increase after a month

A nurse is reviewing the medication administration record of a client who has hypocalcemia and a new prescription for IV calcium gluconate. The nurse should identify that which of the following medications can interact with calcium gluconate?

Digoxin (increased toxicity)---Digoxin induces an increase in intracellular sodium that will drive an influx of calcium in the heart and cause an increase in contractility. "inc. calcium....inc.contraction of dig"

Drugs used to treat hypertension

Diuretics, beta-blockers, calcium channel blockers, angiotension-converting enzyme (ACE) inhibitors, and angiotension II receptor blockers *Clonidine---sedative and Antihypertensive drug; It can treat high blood pressure (A/E dry mouth)

Nurse is caring for older adult pt who has new prescription for zolpidem at bedtime to promote sleep. Nurse should plan to monitor pt for which of following adverse effects?

Dizziness Zolpidem (for the PM) can cause dizziness and daytime drowsiness. It can cause confusion in the older adult client.

Nurse is teaching pt who has new prescription for docusate sodium about med's MOA. Which of folliwng information should nurse include in teaching?

Docusate sodium reduces the surface tension of the stools to change their consistency. Docusate sodium is a surfactant that softens stool by reducing surface tension, allowing water to penetrate more easily into the stool. Osmotic laxatives, such as glycerin suppositories, act by lubricating the lower colon and initiating reflex contractions of the rectum. Bulk-forming laxatives, such as methylcellulose, mimic the action of dietary fiber, forming a viscous compound that softens the fecal mass and increases its bulk, which stimulates peristalsis.Stimulant laxatives, such as bisacodyl, stimulate the intestinal wall to cause peristalsis by pulling water into the intestines.

Donepezil and Rivastigmine

Donepezil and Rivastigmine are medications that treat Alzheimer Disease by increasing the acetylcholine levels in the body "PNS". Although they are not a cure or prevention for Alzheimer disease, they can slow the progression of the disease. Note: choline helps w/ myasthenia gravis also!!

Dopamine vs. Dobutamine

Dopamine is a vasopressor "alpha 1 receptors" lining of smooth muscles vessels "vasopressor" and an inotrope Dobutamine is an inotrope "beta-1 receptors" BUT (-)-dobutamine cab be both a vasoconstrictor, and (+)- a vasodilator depending on the amount

An elderly male client has been taking doxazosin 2 mg daily for 4 weeks for treatment of benign prostatic hypertrophy. The client reports feeling dizzy. The nurse should first: test his urine for ketones. review his other medications. report the symptoms to the health care provider. take his blood pressure lying, standing, and sitting.

Doxazosin is also used as an antihypertensive agent; the client may be experiencing orthostatic hypotension. Because an adverse effect of doxazosin is orthostatic hypotension, the nurse should first take the client's blood pressure; later, she can review other mediations. The client's report of symptoms should be reported to the physician with the blood pressure readings. (less)

Nurse is providing teaching to pt who has prescription for trimethoprim/sulfamethoxazole. Which of following instructions should nurse include in teaching?

Drink 8 to 10 glasses of water daily. MY ANSWER---The nurse should instruct the client to increase water intake to 1,920 to 2,400 mL (65 to 81 oz) a day to decrease the chance of kidney damage from crystallization. The nurse should instruct the client to take the medication on an empty stomach either 1 hr before or 2 hr after meals. The nurse should instruct the client to notify the provider if a rash develops, because this can be an indication of Stevens-Johnson syndrome----- the client should not expect to have a fine, red rash as a transient effect. The nurse should inform the client to store trimethoprim/sulfamethoxazole in a light-resistant container at room temperature.

Tamoxifen

Estrogen modulator---- treats breast cancer. It may also prevent breast cancer in women at high risk of developing it. The client is at risk for gastrointestinal irritation (GI) as an adverse effect of tamoxifen; The client is at risk for hot flashes as an adverse effect of tamoxifen; The client is at risk for vaginal dryness as an adverse effect of tamoxifen;

A nurse is providing teaching to a client about the adverse effects of sertraline. Which of the following adverse effects should the nurse include? A. Excessive sweating B. Increased urinary frequency C. Dry cough D. Metallic taste in mouth

Excessive sweating

Marijuana (Cannabis)

FDA has not approved a marketing application for cannabis for the treatment of any disease or condition and thus has not determined that cannabis is safe and effective for any particular disease or condition.

Nurse is teaching pt who has gout and new prescription for allopurinol. Nurse should instruct pt to d/c med for which of following adverse effects?

Fever Fever can indicate a potentially fatal hypersensitivity reaction. The client should discontinue the allopurinol and notify the provider if a fever or rash develops. ."think PeglotiCASE vs ALLopurinol.....you ALL got a Case with Chronic gout"

Antidote for benzodiazepines

Flumazenil (Romazicon) Go out often in the Benz---liable to catch the FLU!

Common meds in pregnant woman

For high blood pressure>>labatelol, hydralazine, nifedipine • No history of stroke .....high-risk women can take ASA "aspirin" 81mg daily History of stroke, multiple regimens are available; all include Heparin

Sumatriptan (Imitrex)

For migraine ischemic heart disease is a contraindication b/c it constricts blood vessels

Hypomagnesemia (1.3-2.2) VS Hypermagnesemia

Hypomagnesemia (1.3-2.2) ----irritability and tremors Hypermagnesemia----DISORIENTATION and CONFUSION---we give patients in the hospital magnesium drips for pre-eclamapsia and to help baby brain for pre-mature infants--- we have to put mom on fall risk and it surpasses the nervous system!!!------This client is at risk for RESPIRATORY DEPRESSION and CARDIAC DYSRHYTHMIAS because a magnesium level of 3.1 mEq/L is above the expected reference range of 1.3 to 2.1 mEq/L>>>>>>Adminsters The nurse should expect to administer IV CALCIUM GLUCONATE to help the HEART and prepare to provide ventilatory support.

Alendronate (Fosamax)-----drug class bisphosphonate "onate"

For postmenopause Bone health-----inhibits osteoclast activity; It can treat or prevent osteoporosis. It can also treat Paget's disease of the bone; prolonged use of corticosteroids promote osteoclast activity *take on empty stomach--- foods decrease absorption *remain in a sitting position for 30min to prevent ESOPHAGITIS---Alendronate can damage the esophagus both by toxicity from the medication itself and by nonspecific irritation secondary to contact between the pill and the esophageal mucosa, similar to other cases of "pill esophagitis." *Take with a full glass 8oz of water to prevent esophagitis----pill causes severe irritation as its being swollowed *Teach patient to increase intake of calcium and vitamin D so drug can be effective while engaging in weight bearing excercises to help calcium absorption *monitor bone density regularly during therapy *long term A/E IV= Jaw necrosis

Methotrexate

GOLD STANDARD for inflammatory disorders; treat rheumatoid arthritis and psoriasis.-- dose is smaller than treatments for cancer---Chemotherapy and Immunosuppressive drug It can treat cancer of the blood, bone, lung, breast, head, and neck. 1. immunosuppressant and can cause bone marrow supression.....infection risk 2. teratogenic---avoid pregnancy after 3 months its discontinued can cause congenital abnormalities and fetal death 3. hepatotoxic--- avoid alcohol as this an inc. its A/E

Omeprezole (Prilosec)

Gastric Antisecretory - Proton Pump Inhibitor (PPI) H. pylori-caused ulcers are treated with a combination of antibiotics and an acid-reducing proton pump inhibitor. Omeprazole is a well studied and well tolerated agent effective in adults or children as a component in regimens aimed at eradicating H. pylori infections

A nurse is assessing a client who has schizophrenia and is taking haloperidol (1st generation). The nurse should report which of the following findings to the provider as a manifestation of neuroleptic malignant syndrome (NMS)? Haloperidol-- Antipsychotic---It can treat certain types of mental disorders.

HALO--- for inc. temp, Respiratory Distress, "medical emergency" *Temperature of 39.7 (103.5)- FEVER---acute and life threatening emergency *Diaphoresis *Respiratory Distress *hyper/hypotension *incontinence *tachycardia *muscle rigidity

Nurse is caring for pt who has HF and prescription for enalapril. Nurse should monitor pt for which of following as adverse effect of med?

Hyperkalemia due to potassium retention by the kidneys Enalapril "vasodilator" is an ACE inhibitor that has several cardiovascular adverse effects including hypotension, tachycardia, and dysrhythmias. Enalapril can cause several sensory adverse effects such as a loss of taste. However, it does not cause a loss of smell.

hyperkalemia vs hypokalemia

Hyperkalemia---- Hypokamia------.manifestations of hypokalemia such as difficulty concentrating, shallow respirations, hyporeflexia, and muscle weakness---- report them to the provider.

Nurse is caring for pt who has HF and prescription for enalapril. Nurse should monitor pt for which of following as adverse effect of med?

Hyperkalemia----Enalapril improves cardiac functioning in clients who have heart failure and can cause hyperkalemia due to potassium retention by the kidneys. Enalapril tends to raise the serum potassium level and reduce the glomerular filtration rate (GFR). Monitoring the serum potassium and creatinine levels and the GFR is therefore imperative. ACE inhibitors decrease aldosterone production, resulting in reduced sodium and water retention; this also helps lower blood pressure. Enalapril is an ACE inhibitor that has several cardiovascular adverse effects including hypotension, tachycardia, and dysrhythmias, several sensory adverse effects such as a loss of taste. However, it does not cause a loss of smell.

spinal anesthesia does what to BP?

Hypotension due to venous dilation, which may cause tachycardia Nursing interventions if provider agrees: 1. Lower head of bed to allow for adequate venous return 2. Increase fluids 3. administer vasopressors

A nurse at an urgent care clinic is collecting a history from a female client who has a UTI. The nurse anticipates a prescription for ciprofloxacin. The nurse should identify that which of the following client statements indicates a contraindication for administering this medication?

I have tendonitis, so I haven't been able to exercise (risk of tendon rupture)--- this is an A/E of the drug Taking ciprofloxacin increases the risk that you will develop tendinitis (swelling of a fibrous tissue that connects a bone to a muscle) or have a tendon rupture (tearing of a fibrous tissue that connects a bone to a muscle) during your treatment or for up to several months afterward.

A nurse is providing discharge instructions to a client who has heart failure and a new prescription for captopril. Which of the following client statements indicates an understanding of the teaching?

I should tell my provider if I develop a sore throat "April has all A's walking across the vasodilated red Karpet (drug inc. potassium) who has to pee (low aldosterone secretions, Sodium and fluid loss) and her heart is racing "tachycardia" due to possible hypotension, she has a persistent dry cough and just found out she was diagnosed with pancytopenia.......she thinks the low neutrophils are causing her cough but its actually the substance P" (neutropenia is an A/E of Captopril)--Agranulocytosis and neutropenia have been reported for ACE inhibitors,

A nurse is instructing a client on the application of nitroglycerin transdermal patches. Which of the following statements by the client indicates an understanding of the teaching?

I will take the patch off right after my evening meal (med free time 12-14 hr to avoid tolerance)

A nurse is collecting a medication history from a client who has a new prescription for lithium. The nurse should identify that the client should discontinue which of the following over the counter medications?

Ibuprofen (NSAID increase Lithium level)-----also cannot have too much Na missing----low NA inc lithium someone is getting lit and sweating out Na, while also on NSAIDS cause of leg pain----lithium levels are severely increased

A nurse is assessing a client who is taking propylthiouracil for the treatment of Graves' disease. Which of the following findings should the nurse identify as an indication that the medication has been effective?

Increase in ability to focus

A nurse is teaching a client who has a new diagnosis of Parkinson's disease about how levodopa carbidopa can help control symptoms. The nurse should identify that the drug has which of the following pharmacologic effects?

Increases available dopamine in the brain

Dinoprostone Misoprostol pouch

It can help dilate "rippen" the opening of the uterus (cervix) in pregnant women. -patient should lie on back for at least 2 hours w/o getting up using the pouch -pouch lasts for 12hrs and is removed when active labor occurs - Dinoprostone gel is inserted by catheter + usually requires 2 to 3 doses during 12hr time frame

Lactulose

It can treat constipation---Laxative It can also treat liver disease----Ammonia reducer; Lactulose is also used to reduce the amount of ammonia in the blood of patients with liver disease "cirrhosis". It works by drawing ammonia from the blood into the colon where it is removed from the body.

Risedronate----drug class bisphosphonate "onate"

It can treat or prevent osteoporosis. It can also treat Paget's disease of the bone. *take med. with a full cup of water *take on empty stomach as food decreases absorption *take in the morning at least 30 min before each meal

herb: kava

Kava kava ("kava" for short) contains substances called kavapyrones. They act much like alcohol on your brain, making you feel calm, relaxed, and happy. The plant is also thought to relieve pain, prevent seizures, and relax muscles. *increases liver enzymes *Reports in the United States and Europe have linked kava with severe liver problems. A study of 31 people in Hawaii who were regular kava drinkers showed a significantly greater elevation of two liver enzymes compared with people who were not kava drinkers.

What med. is given for hypothyroidism?

Levothyroxine (Synthroid)

What sodium channel blocker is used to treat ventricular dysrhythmias?

Lidocain "lower ventricles" Antiarrhythmic and Anesthetic It can treat irregular heartbeats (arrhythmias). It can also relieve pain and numb the skin.

Raloxifene (Evista)

Like estrogen...... which decreases osteoclast activity "osteoporosis" as well as lowers LDL "Think of This of lady named RaloXifene who has s/s of low estrogen (hot flashes, decreased libido, vaginal dryness) but has inc. bone density and inc. risk for clots (DVT/ leg cramps) as if she had high amounts of estrogen......... The X signifies she can't have babies!" ......off label use lowers LDL decreasing risk of MI treats osteoporosis...binds to estrogen receptors which decreases bone resorption and keeps calcium in the bone----women who go through menopause have less estrogen works as a BRCA antagonist by blocking estrogen at the receptor site in breast tissue----protects against breast cancer also

Furosemide (Lasix)

Loop diuretic Waste electrolytes----Block Na and Cl reabsorption in ascending loop; sodium, potassium wasting (Eat foods high in potassium), chloride, agium, water wasting. Can cause dehydration, hyponatremia, hypochloremia, hypotension, ototoxicity, hypokalemia (and hyperglycemia, hyperuricemia, decreased Ca and Mg)- *Admin low dose furosemide therapy med. undiluted IV bolus---40mg over 2min* lab values to monoitor: 1. Complete blood count should be monitored, because furosemide can cause agranulocytosis "low WBC", anemia, leukopenia, and thrombocytopenia "low platelet count". *Thrombin time, PT, and INR (2-3) do not have to be monitored in a client receiving furosemide. A/E--- tinnitus * Can be used w/ renal impairment Used for pulmonary edema, edema caused by liver/cardiac/kidney disease, htn Also used for hypercalcemia related to kidney stone formation Because loop diuretics such as furosemide promote excretion of potassium, the nurse should also monitor serum potassium levels. Potassium (3.5-5) replacement therapy may be necessary to prevent hypokalemia Lithium is for Na, as K is for Dig (0.5-2)----- Can cause dig toxicity due to hypokalemia---patients taking dig. must maintain k. level between 3.5-5.0---dig must bind to K receptor sites ****can cause lithium toxicity (0.5-1.5) due to hyponatremia----lithium competes w/ Na binding sites Antihypertensives have additive effect NSAIDs reduce diuretic effect----do not take ibuprofen for headache Daily weights and I/O -Take early in the day---- not at bedtime to reduce sleep disruption

Opiod DONT'S!!!!

Lorazapam is for seizures and a CNS depressant also---- double negative... DO NOT combined with opioids

Selegiline

MAOI "monoamineoxidase inhibitors"---Antidepressant It can treat Parkinson's disease and depression. A/E- drowsiness (indication of serotonin syndrome; too much ; reverse by stopping administration of antidepressants or dextromethorphan "cough med"; give benzodiazepines "LAM+PAM" to decrease agitation) Serotonin syndrome= 3 A's 1. Activity of neuromuscular junction 2. Autonomic stimulation "sympathetic nervous system" flight or fright 3. Agitation PLUS FEVER---same as NMS (neuroleptic malignant syndrome..... neuroleptic means antipsychotic) TACHYCARDIA--same as NMS for antipsychotic HYPERTENSION--same as NMS for antipsychotic CLONUS +4 REFLEX---NMS has Rigidity DIARRHEA ---NMS inc. Creatine Kinanse "CK" levels--see in rhabdomyolysis, break down of protein--- and WBC "CLOZAPINE antipsychotic"

Phenelzine (Nardil)

MAOI antidepressant food interaction with tyramine foods or SSRI, MAOI, St. Johns---other antidepressant can cause hypertensive crises tyramine foods pepperoni, aged cheese, fermented food, pepperoni, anything smoked

A nurse is administering the cyclophosphamide orally to a school-age child who has a neuroblastoma. Which of the following actions should the nurse take when administering this medication? Give an antiemetic 30 min after medication administration. Monitor blood glucose levels. Maintain hydration with liberal fluid intake. Monitor for tumor lysis syndrome.

Maintain hydration with liberal fluid intake. Rationale: The nurse should offer fluids frequently to maintain hydration and prevent hemorrhagic cystitis, which is an adverse effect of this medication.

a/e of calcium channel blockers

Monitor vital signs, especially heart rate (bradycardia) and blood pressure (hypotension)----Teach patient how to do this as well and to record it and report to the doctor if heart rate or blood is too low. Risk for reflex tachycardia due to hypotension (this occurs when the blood pressure drops and the heart rate increases to compensate for the decrease in blood pressure)--Teach about orthostatic hypotension and that the patient needs to change positions slowly (this helps when there is a decrease in blood pressure when going from a sitting or lying position to a standing position). Monitor EKG closely for arrhythmias: 1st degree AV block is a side effect due to the decrease in the speed of conduction by the AV node----These medications are contraindicated in patients with 2 or 3rd degree heart block. Monitor for signs and symptoms of heart failure...WHY? Remember these medications (especially the non-dihydropyridines) alter how the heart pumps (creates a negative inotropic effect) and this weakens the heart's contractions-----Signs and symptoms of heart failure: crackles, edema, dyspnea, weight gain...monitor intake and output, daily weights, lung sounds etc. Teach patient to avoid taking CCBs with grapefruit juice....it will increase the drug level of the calcium channel blocker. Encourage a high fiber diet due to constipation with Verapamil or Diltiazem (these medications can slow down GI motility). Watch out for gingival hyperplasia: enlargement of gums....teach patient about good oral hygiene and regular dental visits (mainly with dihydropyridines ex: amlodipine)

Rota virus

Most common cause of infant diarrhea; has been known to cause intussusception as an A/E so do not give to kids who have a history of intussusception Prevention rotavirus "live attenuated" vaccine given is given in 3 doses at ages 2 months, 4 months, and 6 months

Patient has spasticity due to multiple sclerosis and is prescribed Tizanidine (Zanaflex)

Muscle Relaxant---- Youtuber said was extremely sleepy after taking med An addictive drug A/E= sedation, hepatotoxicity---use in EXTREME caution in a client who has preexisting liver damage.

Is it ok to mix Lantus (named glargine on ATI) "long acting insulin" with any other type of insulin?

NO NEVER!! use separate syringes for administering insulin glargine and NPH "cloudy" insulin Admin. once in 24hrs

(Vicodin) hydrocodone & acetaminophen

Narcotic "opioid" Analgesic Controlled substance High risk for addiction and dependence. Can cause respiratory distress and death when taken in high doses or when combined with other substances, especially alcohol or other illicit drugs such as heroin or cocaine.

A nurse in the post-anesthesia recovery unit is caring for a client who received a non depolarizing neuromuscular blocking agent and has MUSCLE WEAKNESS. The should anticipate a prescription for which of the following medications?

Neostigmine----Cholinesterase inhibitor ----Prevents break down of acetylcholine inc. duration of muscle activity/ strength *Used for myasthenia gravis (muscle weakness)*

Is it ok to have an IV heparin (enoxaparin) infusion with aPTT of 90 sec?

No you must reduce the IV infusion rate!!!! -----aPTT rate must be (60-80 sec)

Nurse is providing teaching to pt who has new prescription for ferrous sulfate. Nurse should instruct pt to take med w/ which of following to promote absorption?

Orange juice------ The absorption of ferrous sulfate "iron" is enhanced by a vitamin C source, such as orange juice.

PhenazoPYRIDINE (Pyridium)

PAIN MED---- Pyridine---- PYRO= pain med used to take out that "burning fire" UTI pain med to treat buring pain sensation *Can cause urine to turn reddish/ orange "burning fire"---expected. Do Not notify provider Common brands: AZO Standard Maximum Strength, Urinary Pain Relief, Pyridium Ciprofloxacin hydrochloride usually used to treat UTI infections along with pain med------ Phenazopyridine *Ciprofloxacin hydrochloride (under drug class fluoroquinolone) ---- has been known to cause suicidal ideations, anxiety, and depression

A nurse is teaching a client who has obsessive‑compulsive disorder and has a new prescription for paroxetine. Which of the following instructions should the nurse include?

Paroxetine can take 1 to 4 weeks before the client reaches full therapeutic benefit.-----selective serotonin reuptake inhibitors (SSRIs): TAPER SLOWLY

A nurse is reveiwing the ECG of a client who is receiving IV furosemide for heart failure. The nurse should identify which of the following findings as an indication of hypokalemia?

Presence of U waves= hypokalemia and flat or inverted T-waves

A client is taking Albuterol for asthma and just started taking propranolol (beta-blocker) and reports asthma med is less effective, why?

Propranolol blocks the beta2 receptors in the lungs propranolol (beta-blocker) interferes with albuterol receptors----causes inhibitory interaction*

medication used to treat hyperthyroidism?

Propylthiouracil--for pregnant woman in 1st trimester methimazole----for pregnant woman in 2nd + 3rd trimester Antithyroid agent----It works by making it harder for the body to use iodine to make thyroid hormone. It does not block the effects of thyroid hormone that was made by the body before its use was begun. It can treat Graves' disease and too much thyroid hormone (hyperthyroidism) in patients who have already been treated with medications such as methimazole that did not work well. A/E- hepatotoxic and can cause sever liver injury. Instruct client to report dark urine and yellowing of the eyes indicating jaundice---liver damage Common side effects that does not warrant reporting: 1. Rash is the most common and does NOT indicate allergic reaction 2. Must take drug multiple times each day--- more than once a day 3. may cause drowsiness

Insulin peak times NCLEX Give food during peak times to avoid hypoglycemia----must check glucose levels; Awake ask them to eat; Sleeping= Stab with IV D50; Always Reassess Glucose levels below 70= hypoglycemia (cold and clammy) Rotate location q. 2 weeks; best place is in the abdomen Give insulin on sick days--yes you can still give insulin w/o food just monitor closely; as a matter of fact u will increase insulin on sick days or stressful situations--stress causes glucose levels to RISE

Ready (rapid), Set (short), Inject (intermediate), Love (long) Peaks require plates!! Rapid acting insulin= Humalog, Novalog, Aspart "move ur ass parts its fast acting", Lispro "Lis is a pro she is really fast", Glulisine (Apidra)--Glulisine "goes fast like a limosine give during meals/ food must be at bedside or patient should be eating same time drug is given or within 10 min"; Most Deadly; Onset- 15min Peak- 1hr Duration- 3 hrs ""15 minutes feels like an hour during 3 rapid responses." Short acting insulin= Regular insulin- Humulin R, Novolin R---IV, IVP ONLY!! (vip for the IV lol)---usually mixed w/ intermediate insulin "NPH/ Cloudy" Onset: 30 minutes Peak: 2 hours Duration: 3-6 hours "Short-staffed nurses went from 30 patient to (2) 3-6 patients." INtermediate= Humulin N; Novolin N; NPH "cloudy"; Cloudy will NEVER be put into an IV or IV BAG; Always mix clear to cloudy; Given 2x day Onset: 2 hours Peak: 8 hours Duration: 16 hours "(N)urses (P)lay (H)ero to (2) eight 16 year olds." Long acting= Levemir + Lantus "Lantern burns all night" (glargine "Large Lasting") + Detemir "lasts all year" + DO NOT MIX--use diff. syringe No Peaks so not food dependent---No Plates Give 1x in 24hrs Onset: 2 hours Peak: NONE Duration: 24 hours---give 1x a day "Long acting starts in 2 hours, never peaks, and lasts 24hrs"

Nurse is preparing to mix and administer dantrolene "muscle relaxant" via IV bolus to pt who had developed MH "malignant hyperthermia" during surgery usually triggered by succinylcholine (another type of muscle relaxant). Which of following actions should nurse take?

Reconstitute the initial dose with 60 mL of sterile water without a bacteriostatic agent. The nurse should dilute the medication with 60 mL of sterile water without a bacteriostatic agent and inject rapidly.

2 Types of Calcium Channel Blockers

Relaxes arterioles and slows HR *Dihydropyridines: end with "pine"---Mainly for hypertension and angina; More vascular smooth muscle selective Amlodipine Felodipine Nifedipine *Non-dihydropyridines: more myocardium selective---Antiarrhythmic (helpful with supraventricular tachycardia and atrial fibrillation)----ALONG WITH hypertension, angina Phenylalkylamines: (Verapamil) Benzothiazepines: (Diltiazem) ----Other disease it can treat include reynauds syndrome, cerebral spasms

Antibiotic Drugs used to treat Tuberculosis?

Rifampin + Ethambutol "Eye loss of red/green" + Isoniazid "RIPE" R- Rifampin "Red faminpin"= orange & red tears, sweat, saliva I- Isoniazid "most tested" (INH)= (I) Interferes with the absorption of B-6 (pyridoxine) low B6 causes peripheral neuropathy/ patient must take vitamin B6---not folic acid, B12 or vitamin D! (N) Neuropathy--report new numbness, tingling extremities, ataxia (H) hepatotoxicity--report asap-- jaundice, yellow skin/eyes, dark urine/ elevated liver enzymes AST/ALT; no ETOH alcohol; reduce acetaminophen P- Pyrazinamide "nice to know but not need to know" E- Ethambutol- Eye--- report blurred vision and color changes "loss or red/green color discrimination", frequent eye exams 1. meds lasts 6-12 months 2. N-95 masks worn @ all times 3. family must be tested for TB 4. Sputum sample must be done q. 2-4 weeks 5. 3 neg. cultures on 3 diff. days means no longer infected----3 and then ur free! All TB drugs are hepatotoxic

beta blocker metoprolol used to treat hypertension mask the signs of .............

hypoglycemia Beta-blockers can mask the symptoms of hypoglycemia, such as a rapid heartbeat and tremor because they block the effects of norepinephrine, which results in a slowing of your heart rate and a reduced tremor. Hunger, irritability, and confusion may be concealed as well.

Antibiotic used to treat Tuberculosis?

Rifampin + ethambutol<<<<< "antibiotics"----this is not a corticosteroid and does not cause thrush!! A/E to report to the provider= decreased visual acuity Common A/E--- (red/orange "red tinged urine", saliva, & tears due to Rifamin) discoloration, anorexia) Rifampin "Red faminpin"= orange & red tears, sweat, salivacauses red/ orange bodily fluids---urine, tears, saliva, and sweat "harmless side effect"----Client states my urine is orange in color and my contact are permanently stained orange/ patient should switch to eye glasses while on treatment--- this is harmless!! No need to contact provider PhenazoPYRIDINE (Pyridium)--*Can cause urine to turn reddish/ orange "burning fire" treats PAIN---expected in the name. Do Not notify provider

SLAM= Anti-inflammatory agents for asthma include?

S- Steriods; for long-term treatment --Beclomethasone (sores in mouth wash teeth q. use to avoid thrush, wash device with tap water 2x/week, can cause sepsis + sickness, sugars may be increased which is normal; oral steroids can suppress growth in children "adrenal supression" that's why inhaled route is standard*); -----Fluticasone "long term inhaled corticosteroid decreasing inflamed airway" -----Budesonide "long term inhaled corticosteroid" used to decrease airway inflammation XXXX------Salmeterol IS NOT a steroid but a long acting beta-2 agonist which increases the risk of asthma related death to decrease the risk med is combined with a inhaled steroid XXXX----- Theophylline long term bronchodilator that alleviates broncospasms by relaxing smooth muscles; "Theo is a God + will see God if taking this med due to toxic effects" toxic + cause tachycardia, do not mix w/ caffeine (10-20 range); Methylxanthines (Aminophylline) L- Leukotriene inhibitors---Monteleukast--kast out mucus production M- Mast cell stabilizers "Cromolyn" histamine stabilizer

long acting beta 2 agonists bronchodilators?

Salmeterol---slower acting--bronchodilator----long acting form of albuterol; *places client for long term ASTHMA related DEATH*; NOT for acute asthma attacks used long term Formoterol Symbicort --this drug is a combination of a long-acting beta agonist AND corticosteroid NOT for an acute asthma attack *DISADVANTAGE of long acting is dosages of med. is fixed

Anti-depressants

Serotonin syndrome (overdose impending death!!) can occur when mixing anti-depressants —no mixing anti-depressants MAOI, SSRI, SSNRI, Tri + tetra- cyclics, St-John worts) Selective serotonin reuptake inhibitors (SSRIs): Paroxetine*, SERTRALINE, citalopram, escitalopram, fluoxetine, fluvoxamine Serotonin-norepinephrine reuptake inhibitors (SNRIs): Venlafaxine*, duloxetine, desvenlafaxine

A nurse at a clinic is providing follow - up care to a client who is taking fluoxetine (SSRI) for DEPRESSION. Which of the following findings should the nurse identify as an adverse effect of the medication?

Sexual dysfunction---OCCURS IN 70% OF patient suppresses platelet aggregation, which increases the risk of bleeding when used concurrently with NSAIDs and anticoagulants. other s/s: decreased libido, impotence, delayed orgasm,

how to use asthma pump

Shake it, before you take it breath all the way out seal lips, puch inhaler inhale and hold breath for ten sec. after inhaling med--- Med. effective if: 1. there is decrease in RR 2. O2 sat at least 90% or higher Not working after 3 doses notify HCP! Albuterol nebulizer treatment will cause: 1. Inc. in productive cough 2. reports of decreased anxiety 3. Mild bilateral hand tremors If taking albuterol with steroid asthma pump...take albuterol 1st to get all that powder down into the lungs! For albuterol Clean mouth piece 1-2x per week with warm water; for steroid inhaler wash after q. use!

Trimethoprim / Sulfamethoxazole

Sulfamethoxazole and trimethoprim combination is used to treat infections including urinary tract infections, middle ear infections (otitis media), bronchitis, traveler's diarrhea, and shigellosis (bacillary dysentery).

A nurse is providing teaching to a client who has a prescription for ergotamine sublingual to treat migraine headaches. Which of the following information should the nurse include in the instructions?

Take one tablet at onset of migraine

Taking digoxin with calcium channel blocker "CCB"?

Taking Digoxin too? Digoxin levels (0.5-2) have to be monitored because CCBs can increase digoxin levels. "Remember Digoxin is toxic when he has low mag, potassium, but inc. calcium channel blockers and inc. calcium" Electrolyte disturbances such as hypomagnesemia, hypercalcemia, and hypokalemia lead to increased sensitivity to digoxin making toxicity more likely even with a lower concentration of serum digoxin. Watch out for Digoxin toxicity.....signs and symptoms include (teach these signs and symptoms to the patient too): EARLY signs and symptoms are GI-related: Nausea, vomiting, anorexia Other signs and symptoms: vision changes with reports of yellow-greenish vision or halos, blurred vision and EKG changes....(dysrhythmias)...tends to be later on.

Oral drug for asthma

Theophylline: given PO not as common because of possible toxicity and maintaining blood levels for this drug is 10-20 mcg/mL AVOID consuming products with caffeine while taking this medication...WHY? Caffeine has the same properties as Theophylline, which can increase the toxic effects of the medication.

Asthma meds given in combination-- Is it ok to give Theophylline with zileuton?

Theophylline= Bronchodilator--- Theophylline May cause--- "Theo is a God + will see God if taking this med due to toxic effects" toxic + cause tachycardia, do not mix w/ caffeine (10-20 range)- Zileuton--*Leukotriene Modifiers (oral): HEPATOTOXIC "monitor ALT* "Montelukast"+ "Zileuton"--Anti-inflammatory "long term" When this function is blocked it leads to the relaxation of the smooth muscle and decreased mucous production...NOT for an acute attack While Theophylline Bronchodilates by relaxing smooth muscle, while Zileuton block leukotrienes stopping inflammation "anti-histamine"----ZILEUTON *Leukotriene Modifiers impairs metabolism of certain meds causing toxicity when combines---Theophylline can inc. in the body becoming toxic A/E of Theophylline include tachycardia

Nurse is reviewing medical record of pt who has schizophrenia and prescription for chlozapine. Which of following lab tests should nurse review before administering med?

Total cholesterol The nurse should review the client's total cholesterol before administering clozapine, because this medication can cause hyperlipidemia. "think of Schizophrenic person "ChlorPROMaZine" at prom with Amantadine "Amanda" as his date who is his antidote and decreases his extrapyradimal effects; Chlorpramazine has a twin who also has extrapyradimal effects named Clozapine who also has Schizophrenia and likes to hide in the cloZet..... he is severely sick (agranulocytosis, anticholinergic effects, high cholesterol, diabetes mellitus, Myocarditis (has a big heart), Oculogyric crisis "rolling back of the eyes", Hormonal side effects)"

A nurse is caring for a client who is in labor. The client is receiving oxytocin by continuous IV infusion with a maintenance IV solution. The external FHR monitor indicates late decelerations. Which of the following actions should the nurse take first? 1. Turn the client to a side lying position 2. disconnect oxytocin from IV 3. Apply oxygen face mask 4. inc. client maintenance infusion IV

Turn the client to a side lying position (reduce risk of uteroplacental insufficiency)---- OXYGENATION to baby is priority since fetus is the one getting O2 cut off!!

Fludrocortisone (Florinef)

a corticosteroid, is used to help control the amount of sodium and fluids in your body. Treat adrenal insufficiency (Addison's disease)---low mineralocorticoids "excessive amounts of sodium are lost in the urine"., glucocorticoids, adrenocorticoids<<<<patient must carry a supply of emergency pills or single use injectable for times of stress-- since body cannot produce its own glucocorticoids to burn fuel for inc. metabolism

Patient-controlled analgesia (PCA)

a drug delivery system that uses a computerized pump with a button the patient can press to deliver a dose of an analgesic through an intravenous catheter -pump is set to deliver a certain amount within a time frame---pump will not deliver more than the programmed amount! - patient should self-administer pain med 10 min B4 activity to reduce discomfort -Pump is set to deliver a larger continuous does when patient is sleeping to ensure comfort while at rest -provide increased fluid while using pump b/c opioids like morphine can cause constipation-----Oral morphine takes atleast an HOUR to work -Encourage early ambulation while using PCA post-op---advise client to sit on side of bed before standing to prevent orthostatic hypotension (DO NOT maintain client on bed rest)

Disulfiram

a medication used to support the treatment of alcohol use disorder by producing an acute sensitivity to ethanol (drinking alcohol). Disulfiram works by inhibiting the enzyme acetaldehyde dehydrogenase, causing many of the effects of a hangover to be felt immediately following alcohol consumption. It's used in recovery programs that include medical supervision and counseling.

A nurse is providing teaching about lithium to a client who has bipolar disorder. Which of the following statements should the nurse include in the teaching? a. "Expect to have blurred vision while taking this medication" b. "Notify your provider if you experience increased thirst." c. "You might be unable to have an orgasm while taking this medication" d. "You should take this medication on an empty stomach"

a. "Expect to have blurred vision while taking this medication" i. Blurred vision is a manifestation of lithium toxicity. The nurse should instruct the client to discontinue the medication and notify the provider if blurred vision occursb. "Notify your provider if you experience increased thirst." i. The nurse should recognize that an increase in thirst is a manifestation of lithium toxicity. The nurse should instruct the client to report increased thirst vomiting, diarrhea, or tremors to the provider c. "You might be unable to have an orgasm while taking this medication"i. Anorgasmia is an adverse effect of some antidepressants. However, it is not an adverse effect of lithiumd. "You should take this medication on an empty stomach" i. Lithium is irritating to the lining of the stomach and increases the client's risk for gastrointestinal upset. The nurse should instruct the client to take the medication with meals

Medroxyprogesterone

brand name Depo-Provera among others, is a hormonal medication of the progestin type. It is used as a method of birth control and as a part of menopausal hormone therapy. brand name Depo-Provera among others, is a hormonal medication of the progestin type. It is used as a method of birth control and as a part of menopausal hormone therapy. 1st dose should be given at 6 weeks postpartum as long as the client is ONLY breast feeding (1st dose should be administered within the first 5 days postpartum only if the client is NOT breast feeding AND ensuring client is not pregnant!); 2nd shot administered 3 months after the first dose

Epinephrine

adrenaline; SNS a hormone and neurotransmitter used to treat: -allergic reactions---peripheral vasoconstriction -to restore cardiac rhythm---flight or fright "inc. cardiac output" -to control mucosal congestion/ asthma--- "bronchodilation, reduces secretions" -glaucoma----reduce intraocular pressure by decreasing aqueous formation "reduces secretions" and increasing outflow facility. *injected via several routes----subcutaneous is more concentrated that IV, since its not directly inserted into the circulatory system. IV concentration is diluted SubQ (concentrated) concentration inserted IV can cause severe tachycardia, peripheral vasoconstriction, hypertension, cerebral hemorrhage, stroke, + death. Note the Below Factors DO NOT effect IV admin: 1. reversibility of med admin---its too late its already in blood stream 2. potential barriers---there r non its put directly into blood stream 3. gastric emptying ---- it's not a pill!

Ergotamine

allows cerebral constriction 1. drug is addictive--- patient should take med. according to the prescribed dose and only take as needed ---patient can have signs of withdraw= headache, nausea, vomiting, restlessness 2. Not safe during pregnancy and can stimulate uterine contractions 3. Tingling in the fingers is signs of decreased of perfusion "drug promotes constriction", also known as too much of the drug "ergotism/ toxicity"

Gentamicin (Garamycin)

aminoglycoside antibiotic When to obtain peak levels for this amino glycoside antibiotics--- 1 hour after administration of IM injection or 30min for IV infusion Obtain trough level immediately before med. admin

epoetin alfa (Epogen, Procrit)

anemia-----used to treat anemia (a lower than normal number of red blood cells) in people with chronic kidney failure!! (condition in which the kidneys slowly and permanently stop working over a period of time). ----kidneys secrete erythropoietin allowing bone marrow to produce RBC's A/E----hypertension, teaches, seizures, heart failure, thromboembolotic events due to inc. hemoglobin levels

Amphotericin B

anti-fungal. HIGHLY TOXIC - infusin rxns (fever and chills)----med like Tylenol, metic given before the infusion--- expected not reason to stop infusion - nephrotoxicity---monitor BUN, creatinine - hypokalemia---patient will need potassium supplementation - hepatoxicity - gynecomasita - C/I with aminoglycosides (just like PCN)

Gabapentin

anticonvulsant and Neuropathic PAIN---Nerve pain medication (most common lie cancer in the lumbar vertebrae_ It can treat seizures and pain caused by shingles. do not stop drug abruptly---- can have withdraw effects "tremors"

Lamotrigine

anticonvulsant/mood stabilizer It can treat seizures and bipolar disorder. A/E- 1. Diplopia 2. Rash---PRIORITY as this is Steven-johnsons syndrome showing for the 1st 2-8 weeks, this inflammatory response can kill! 3. Dizziness 4. Headache

Katamine

antidepressant for SEVERE DEPRESSION + pain med------The only one that the FDA has approved as a medication for depression side effects: nausea (ondansetron given prior to admin), vomitting, hallucinations It's sometimes used illegally by people to get high. routes: IV, IM, Intranasal

Vasopressin

antidiuretic hormone-- Treats: 1. diabetes insipidus 2. cardiac arrest--- increase blood pressure in patients with vasodilatory shock 3. Asystole is when your heart's electrical system fails, causing your heart to stop pumping---not enough blood flow to hear inc. blood pressure. This is also known as "flat-line" or "flat-lining" because it causes your heart's electrical activity to look like a flat line on an electrocardiogram

Diphenhydramine (Benadryl)

antihistamine----causes drowsiness---1st generation med. Anti-histamines can be non-sedating-- options---2nd generation antihistamines: Fexofenadine (Allegra®) Cetirizine (Zyrtec®)

a nurse is teaching client about raloxifene therapy to prevent osteoporosis. the nurse should instruct the client to monitor for which of the following indications of an adverse reaction to this drug?

calf pain ----DVT "Think of This of lady named RaloXifene who has s/s of low estrogen (hot flashes, decreased libido, vaginal dryness) but has inc. bone density and inc. risk for clots (DVT/ leg cramps) as if she had high amounts of estrogen......... The X signifies she can't have babies!"

What is the 1st line drug used for Acute withdrawal?

chlordiazepoxide-----Sedative It can treat anxiety, alcohol withdrawal symptoms, and tremor. *For clients who are nauseated or vomiting, another benzodiazepines (lorazepam) can be administered IV

Hydrochlorothiazide (Microzide)

diuretic; treats hypertension, heart failure, DIABETES INSIPIDUS!!! Note: Thiazides have been used in patients with nephrogenic diabetes insipidus (NDI) to decrease urine volume, but the mechanism by which it produces the paradoxic antidiuretic effect remains unclear.

Amlodipine has been administered for hypertension, state A/E?

dizziness palpitations peripheral edema patient must avoid activities that require alertness

Pramipexole (Mirapex)

dopamine agonist; Dopamine promoter "The guy who has Parkinson's takes Pramipexole to decreases his tremors......but now has hallucinations from taking drug" It can treat Parkinson's disease and restless legs syndrome and tremors

Sucralfate (Carafate)

gel-like substance that lines the stomach lining to protect it from HCL acid when an ulcer is present

magnesium hydroxide

in a class of medications called saline laxatives. It works by causing water to be retained with the stool. This increases the number of bowel movements and softens the stool so it is easier to pass.

Furosemide (Lasix)

loop diuretic *may cause ototoxicity --- patient should tell provider if tinnitus, full feeling in the ears, or hearing loss occurs

Gemfibrozil

lowers high cholesterol and triglyceride levels in the blood by decreasing the liver's uptake of fatty acids A/E liver toxicity*----jaundice

where are Beta-1 receptors found? -----ends in lol

mainly HEART but also kidneys in juxtaglomerulos cell which secrete renin (NA, Water, and vasoconstriction)

where are Beta-2 receptors found? ----ends in lol

mainly in the lungs, but also: 1. skeletal muscle, smooth muscle, digestive system "liver", ciliary body of the eye "ring shapes muscle that change shape of the eye to help it focus"

Cromolyn

mast cell stabilizer----This drug prevents the release of inflammatory chemicals such as histamine from mast cells---stops histamine release (guy riding in Crome hist's at the ladies to calm down their histamine "in heat" release) *used for asthma before exercise prophylactically on a fixed schedule --- not for acute asthma attacks *also used to treat seasonal allergies or rhinitis "inflammation of nose/ stuffy nose" *Use as long term aid for asthma, not for acute asthma attacks bronchospasm----cromolyn therapeutic effects may take several weeks before clinical response is witness 1. Taper of slowly----if abruptly stopped may cause rebound broncospasms NSAID that inhibits the IgE mediated release of antiflammatory mediators from mast cells

A nurse is providing teaching to a client who has a prescription for buspirone. Which of the following instructions should the nurse include?

medication has a low risk for dependency---cause you can ride the bus with buspirone

which of the following is a therapeutic action of raloxifene?

mimics the effects of estrogen on bone tissue----calcium absorption into bone---prevents osteoporosis "Think of This of lady named RaloXifene who has s/s of low estrogen (hot flashes, decreased libido, vaginal dryness) but has inc. bone density and inc. risk for clots (DVT/ leg cramps) as if she had high amounts of estrogen......... The X signifies she can't have babies!"

Vitamin A deficiency

night blindness------- Carrots are an excellent source of vitamin A, with one serving providing 184% of your daily value, and also the following nutrients: Vitamin C. Calcium. Iron.

Nitroglycerin

nitrate drug used in the treatment of angina, AMI, CHF max of 3 doses sublingually Apply ointment using dose measuring applicator + cover with plastic wrap + use gloves + do not rub/massage A drug that helps to dilate the coronary vessels that supply the heart muscle with blood. not if on erectile dysfunction meds not if in cardiogenic shock dont give if hypotension is present side effect is headache ---may cause ST changes on the ECG

isosorbide mononitrate

nitrate with vasodilating properties. It is an anti-anginal agent that works by relaxing the smooth muscles of both arteries and veins, but but predominantly veins to reduce cardiac preload. *due to vasodilation properties it may cause orthostatic hypotension---patient should change positions slowly medication used for heart-related chest pain, heart failure and esophageal spasms. It can be used both to treat and to prevent heart-related chest pain; however, it is generally less preferred than beta blockers or calcium channel blockers

A nurse is assessing a client who is taking amiodarone to treat atrial fibrillation. What finding is a manifestation of amiodarone toxicity?

note: Antiarrhythmic--- used to suppress ventricular and supra ventricular tachyarrhythmias Productive cough--- "picture guy Amiodarone with blue face coughing (pulmonary toxicity) and is putting on sunscreen to decrease his photosensitivity" Pulmonary toxicity is one of the most serious adverse effects of amiodarone, the most common skin side effects is photosensitivity. Skin turns a blue-gray color when exposed to sunlight (sun screen is recommended) a/e photosensitivity pigmentation of the skin pulmonary fibrosis; productive cough peripheral neuropathy

Paroxetine

obsessive‑compulsive disorder ; Anxiety Disorders

Naloxone (Narcan)

opiate antagonist, reverses opioid induced resiratory depression

Pregabalin

pain med. for fibromyalgia----nerve pain/ muscle pain Common brands: Lyrica It can also treat seizures.

PCA pump

patient controlled analgesic administered intravenously with a machine *Must have second nurse check PCA settings

Asthma s/s

prolonged exhalation causing acidosis (PaO2 less than 80, PH below 7.35, PaCO2 over 45, Elevated Eosinophils= Allergic reaction--not neutrophils or lymphocytes---EEWW think mucus and closing bronchioles Causes/ triggers 4 asthma attacks- *stress, sickness, sever weather "cold", strenuous activity "take med 20-30min b4 exercise", *NSAID drugs (NAPROXEN, salycilate acid "aspirin", Ibuprofen, Indomethacin, KETOROLAC) CAPS ON NCLEX *Beta-blockers- propanolol must be non-selective, blocks both---blocks heart rate "Beta-1" and lungs "Beta-2" latex, antibiotics, penicillin do NOT cause asthma attacks 1st "early" sign of hypoxia- mental status change, agitation, restlessness, drowsiness Must know the peak expiratory flow rate: obtain stats 4rm a 2week period when asthma is controlled---there is no daily best score Peak flow rate- inhale deeply, seal lips, exhale fast and strong, 2x with break of 5-10 sec in between; record 1 score of the highest 3 attempt--do no average scores! No change after green-- asthma is 80-100 under control; not further med is needed due to no worsening symptoms yellow-- asthma not under control; need for med admin red; rescue drug used q. 4hrs for 1-2 days; call HCP for additional meds or change in treatment Red---emergency treatment is need immediately after taking rescue drug

lactulose (Cephulac) is given to a patient who has cirrhosis of the liver----(the liver converts ammonia "break down of protein" to urea when diseased ammonia can build up causing encephalopathy + neuropathy) What is the indication of lactulose?

reduce blood ammonia from break dow of protein by excretion of ammonia by stools ---draws water into the stool and softens it ---- MOST COMMON INDICATION: inhibits ammonia from passing into the blood stream, allowing it to be passed out into the stool... this will decrease the amount of ammonia in the blood and as a result reducing neuropathy and hepatic encephalopathy (cause depressed RR, *abdominal distention will not improve with this * loose the ammonia via feces NOT urine *patient will have 2 -3 soft stools per day if its used for constipation; If patient has high ammonia levels they may pass more stool *Diarrhea and loose stools that lactulose is working--- the only indicator that the drug is working is IMPROVED MENTAL STATUS *ammonia levels will decrease and MENTAL STATUS will improve since it is reducing encephalopathy

s/s of benzodiazepines anxiolytics ?

sedation lightness ataxia decreased cognitive function

Clonidine

sedative (aid in anxiety) and Antihypertensive drug Alpha-2-Agonist "blocks SNS" ; It can treat high blood pressure *can cause dry mouth

asthma patient prescribed beta2-agonist bronchodilator (albuterol), what is it's indication?

short acting med. which reverses bronchospasm----fast relief during an attack NOT for daily treatment ---remember inhaled corticosteroid prednfisone is long acting treatment to reduce inflammation

Name the beta-2 selective agonists (4)

short acting--alBUTEROL (prototypical), pirBUTEROL, metaproterenol----short-acting beta-adrenoceptor agonist that is used for the immediate treatment of bronchoconstriction, as a rescue medication for asthma, or as a bronchodilator for chronic airflow obstruction. terbutaline---used as a fast-acting bronchodilator as it activates the sympathetic nervous system (often used as a short-term asthma treatment); .........also used as a tocolytic ( drugs that are used to DELAY NOT stop your delivery for a short time (up to 48 hours) "---- to delay premature labor<<<activating the sympathetic nervous system inhibits contractions of myometrial smooth muscle cells when beginning labor too early in your pregnancy; ...........As an asthma treatment, the inhaled form of terbutaline, starts working within 15 minutes and can last up to 6 hours; ..........Since terbutaline activates the sympathetic nervous system, it can cause tachycardia

ethinyl estradiol/norelgestromin

sold under the brand name Ortho Evra among others, is a CONTRACEPTIVE PATCH containing the progestin norelgestromin and the estrogen ethinylestradiol.

An elderly male client has been taking doxazosin 2 mg daily for 4 weeks for treatment of benign prostatic hypertrophy. The client reports feeling dizzy. The nurse should first:

take his blood pressure lying, standing, and sitting. Doxazosin is also used as an antihypertensive agent; the client may be experiencing orthostatic hypotension. The nurse should first take the client's blood pressure; >Later, the nurse can review other medications. >The client's report of symptoms should be reported to the health care provider with the blood pressure readings.

Asthma med that is a bronchodilator that is used to alleviate bronchospasms by relaxing smooth muscles in the bronchi during asthma attack

theophylline "Theo is a God + will see God if taking this med due to toxic effects" toxic + cause tachycardia, do not mix w/ caffeine (10-20 range); Methylxanthines (Aminophylline)

Zolpidem (Ambien)

think Zolpidem is for the PM Sedative------It can treat insomnia. A/E- *dizziness & daytime drowsiness

A nurse is caring for a client who is taking acetazolamide fr chronic open angle glaucoma. For which of the following adverse effects should the nurse instruct the client to monitor and report?

tingling of fingers

why does a nurse inject medication slowly?

to reduce toxicity and infiltration -CNS toxicity can become evident within 15sec!!! At least if you go slowly so A/E can be much less and severe

calcitonin-salmon

tonin--- tone down calcium in the blood used to treat osteoporosis in women who are at least 5 years past menopause and cannot or do not want to take estrogen *intra nasal----hold bottle upright and depress both white side arms towards the bottle 6x; A/E nasal dryness

Simvastatin (Zocor)

treat hyperlipidemia Statin It can treat high cholesterol and triglyceride levels. This may reduce the risk of heart attack, stroke, and related health conditions. A/E: *REPORT--MUSCLE PAIN and weakness---can cause muscle injury---sever myostitis "muscle inflammation" (a rare group of diseases characterized by inflamed muscles, which can cause prolonged muscle fatigue and weakness. The group includes the autoimmune disorders juvenile myositis, dermatomyositis and polymyositis, as well as inclusion body myositis (IBM). Expected Findings: *Constipation *Flatulence *Headaches

the herb black cohosh uses

treat menopausal symptoms (including hot flashes and night sweats (together known as vasomotor symptoms), vaginal dryness, heart palpitations, tinnitus, vertigo, sleep disturbances, nervousness, and irritability

Ethambutol (Myambutol)

treatment for TB E- Ethambutol- Eye --- report blurred vision, color changes, frequent eye exams

Hydroxyzine

treats anxiety + is a antihistamine do decreases inflammation-----used to help control anxiety and tension caused by nervous and emotional conditions. *A/E- dry mouth

Ramelteon (Rozerem)

treats insomnia ----Sedative.----melatonin receptor agonist

Sumatriptan (Imitrex)

used for cluster headaches---causes cerebroconstriction A/E- 1. CHEST PRESSURE + HEAVHY ARMS----50% of patients experience this and s/s will resolve 2. hypertension

Acetaminophen (Tylenol)

used for mild to moderate pain and fever ONLY!----Acetaminophen is only effective at relieving pain and fever, while ibuprofen relieves inflammation in addition to pain and fever.

Meperidine (Demerol)

used to relieve pain severe enough to require opioid treatment and when other pain medicines did not work well enough or cannot be tolerated. Narcotic It can treat moderate to severe pain. injected 1 to 1 and 1/2 inch for a BMI 23 *500mg PO is way too much; recommended range is 50mg

Rifampin

used to treat tuberculosis.....usually used in combination with Isoniazid A/E 1. causes red/ orange bodily fluids---urine, tears, saliva, and sweat "harmless side effect"----Client states my urine is orange in color and my contact are permanently stained orange/ patient should switch to eye glasses while on treatment--- this is harmless!! No need to contact provider 2. Hepatotoxicity--- most antibiotics 3. Decreases effectiveness of oral contraceptives--- most antibiotics

Isoniazid

used to treat tuberculosis.....usually used in combination with Rifampin Can be used for both active + latent TB + prophylactically to those exposed to TB I- Isoniazid "most tested" (INH)= (I) Interferes with the absorption of B-6 (pyridoxine) low B6 causes peripheral neuropathy/ patient must take vitamin B6---not folic acid, B12 or vitamin D! (N) Neuropathy--report new numbness, tingling extremities, ataxia (H) hepatotoxicity--report asap-- jaundice, yellow skin/eyes, dark urine/ elevated liver enzymes AST/ALT; no ETOH alcohol; reduce acetaminophen *no issues w/ birth control 1. Hepatotoxicity---jaundice, nausea, vomiting, dark colored urine "monitor AST/ALT live enzymes"----cannot take with other hepatotoxic substances like acetomenphene + alcohol---if patient has existing liver damage they cannot take this med! 2. Peripheral neuropathy "damages nerves/ tingling in extremities" ---Isoniazid can interfere with the activity of vitamin B6 (pyridoxine). Vitamin B6 supplementation is recommended, especially in people with poor nutritional status, to prevent development of isoniazid-induced peripheral neuritis (inflamed nerves). *vitamin B6 (pyridoxine)= important for normal brain development and for keeping the nervous system and immune system healthy.

Nitroprusside

vasodilator---very POTENT that works faster than any other med. Available!!!!!----given to clients who need RAPID reduction in blood pressure 1. regulates IV pump using clients weight 2. BLOOD pressure MUST be monitored with an arterial line or q. 15min with electric monitor device 3. change bag q. 24hrs---NOT 48 4. Freshly prepared solution should be LIGHT BROWN----DISGARD if blue, green, dark drown, dark red 5. must cover nitro bag with OPAQUE bag A/E- Delirium *----symptoms caused by cyanide intoxication are non-specific and include metabolic acidosis, headache, seizures, coma, delirium, weakness, dyspnea . Nitroprusside is rapidly hydrolyzed (half-life, 11 minutes) and releases free cyanide,

Niacin (B3)

vitamin B3 that's made and used by your body to turn food into energy. *Reduces LDL cholesterol-----low density floats---good cholesterol has protein * Hepatotoxic--- contraindicated in acute liver disease, peptic ulcer, and presence of any active hemorrhagic process It helps keep your nervous system, digestive system and skin healthy. Niacin (vitamin B-3) is often part of a daily multivitamin, but most people get enough niacin from the food they eat.

name 2 anticoagulants---stops the formation of clots

warfarin (Coumidin), heparin (enoxaPARIN "low molecular HEPARIN) *Prevents GROWTH---of NEW + Existing clots; DO NOT dissolve clots like thrombolytics "tPA + Alteplase" *For patients recovering 4rm an MI "heart attack" or DVT *DO not take NSAIDS with anticoagulants *If possible, warfarin therapy should be avoided during pregnancy. If warfarin therapy is essential, it should be avoided at least during the first trimester (because of teratogenicity) and from about 2 to 4 weeks before delivery to reduce risk of hemorrhagic complications.

A nurse is reveiwing the ECG and patient has hyperkalemia. The nurse should identify which of the following findings as an indication of hyperkalemia?

widened QRS complex "longer contractions" + Tall T waves Regular insulin or rapid acting insulin "lispro" is administered to treat hyperkalemia------ Insulin therapy lowers K+ concentration driving K+ into cells (both directly and indirectly by reversing hyperglycemia). Therefore, insulin therapy may cause severe hypokalemia. IV regular insulin is often used during acute hyperkalemia management due to its quick onset of action and moderate duration of redistribution effect (off-label use) (1 ,2). Insulin 10 units is estimated to lower serum potassium by 0.6-1.2 mMol/L within 15 minutes of administration with effects lasting 4-6 hours (1-3).

Status asthmatics is a medical emergency

will NOT resolve with medication---inflammation so severe that no airflow is possible must do endotracheal intubation

Status asthmaticus treatment? acute severe asthma or a severe asthma exacerbation is the NEWER TERM

will NOT resolve with medication---inflammation so severe that no airflow is possible must do endotracheal intubation Asthma attack med. admin "AIM" A= Albuterol beta 2 agonist short acting dilating bronchioles but is also activates beta- 1 in heart causing tachycardia---BUTEROL= BRUTAL ASTHMA ATTACKS I= Ipratropium ---form of atropine short acting ANTICHOLINERGIC--"when you trop "hop" you go= SNS"----treats bronchospasm for COPD patients by opening up airways M= Methyl-preniso-lone (Solu Medrol) sound like prednisone which is how u know it's a steroid....used last b/c slow acting; Methylxanthines-- Theophylline (bronchodilator)--- "Theo is a God + will see God if taking this med due to toxic effects" toxic + cause tachycardia, do not mix w/ caffeine (10-20 range); Methylxanthines (Aminophylline)---bronchodilator If meds below do not work then this is status asthmatics....It refers to an asthma attack that doesn't improve with traditional treatments, such as inhaled bronchodilators. These attacks can last for several minutes or even hours. epinephrine, potent systemic bronchodilator oxygen IV fluids steroids, *****Always administer the bronchodilator FIRST and then 5 minutes later the corticosteroid. If not responding to treatment, may need intubation and mechanical ventilation.

Nurse is preparing to teach pt who is to start new prescription for extended release verapamil. Which of following instructions should nurse plan to include?

​Change positions slowly.The nurse should instruct the client to change positions gradually to prevent orthostatic hypotension and syncope. "think someone on the stage rapping and artist has his calcium channel blockers protecting him.... he is moving slow to prevent orthostatic hypotension....his song is about foods high in fiber so you do not get constipated.....the calcium channel blockers are fed up of the nifedipine/amlodipine phenes"


Conjuntos de estudio relacionados

Structural & Functional Classification of Joints

View Set

Chapter 27 - Listening Guide Quiz 16b: Mozart: Eine kleine Nachtmusik, III

View Set

CCNA 2 Chapter 5: Switch Configuration

View Set

American History midterm practice

View Set

CompTIA SY0-501 Security+, - Merged RM1

View Set

WQ Chapter 8 - Appendicular Skeleton (Lower Limb)

View Set

Chapter 7: Episodic & Semantic Memory

View Set